Hostname: page-component-8448b6f56d-qsmjn Total loading time: 0 Render date: 2024-04-24T08:35:50.825Z Has data issue: false hasContentIssue false

Inverse K-Chevalley formulas for semi-infinite flag manifolds, I: minuscule weights in ADE type

Published online by Cambridge University Press:  07 July 2021

Takafumi Kouno
Affiliation:
Department of Mathematics, Tokyo Institute of Technology, 2-12-1 Oh-okayama, Meguro-ku, Tokyo152-8551, Japan E-mail: kouno.t.ab@m.titech.ac.jp.
Satoshi Naito
Affiliation:
Department of Mathematics, Tokyo Institute of Technology, 2-12-1 Oh-okayama, Meguro-ku, Tokyo152-8551, Japan E-mail: naito@math.titech.ac.jp.
Daniel Orr*
Affiliation:
Department of Mathematics (MC 0123), 460 McBryde Hall, Virginia Tech, 225 Stanger St., Blacksburg, VA 24061, USA
Daisuke Sagaki
Affiliation:
Institute of Mathematics, University of Tsukuba, 1-1-1 Tennodai, Tsukuba, Ibaraki305-8571, Japan E-mail: sagaki@math.tsukuba.ac.jp.
*
E-mail: dorr@vt.edu

Abstract

We prove an explicit inverse Chevalley formula in the equivariant K-theory of semi-infinite flag manifolds of simply laced type. By an ‘inverse Chevalley formula’ we mean a formula for the product of an equivariant scalar with a Schubert class, expressed as a $\mathbb {Z}\left [q^{\pm 1}\right ]$ -linear combination of Schubert classes twisted by equivariant line bundles. Our formula applies to arbitrary Schubert classes in semi-infinite flag manifolds of simply laced type and equivariant scalars $e^{\lambda }$ , where $\lambda $ is an arbitrary minuscule weight. By a result of Stembridge, our formula completely determines the inverse Chevalley formula for arbitrary weights in simply laced type except for type $E_8$ . The combinatorics of our formula is governed by the quantum Bruhat graph, and the proof is based on a limit from the double affine Hecke algebra. Thus our formula also provides an explicit determination of all nonsymmetric q-Toda operators for minuscule weights in ADE type.

Type
Algebra
Creative Commons
Creative Common License - CCCreative Common License - BY
This is an Open Access article, distributed under the terms of the Creative Commons Attribution licence (http://creativecommons.org/licenses/by/4.0/), which permits unrestricted re-use, distribution, and reproduction in any medium, provided the original work is properly cited.
Copyright
© The Author(s), 2021. Published by Cambridge University Press

1. Introduction

Let $\mathbf {Q}_{G}^{\mathrm {rat}}$ be the semi-infinite flag manifold. This is a reduced ind-scheme whose set of $\mathbb {C}$ -valued points is $G(\mathbb {C}\left (\!\left (z\right )\!\right ))/(H(\mathbb {C}) \cdot N(\mathbb {C}\left (\!\left (z\right )\!\right )))$ (see [Reference Kato11] for details), where G is a simply connected simple algebraic group over $\mathbb {C}$ , $B=HN\subset G$ is a Borel subgroup, H is a maximal torus and N is the unipotent radical of B. For each affine Weyl group element $x\in W_{\mathrm {af}} = W \ltimes Q^{\vee }$ , with $W = \langle s_i \mid i \in I \rangle $ the (finite) Weyl group and $Q^{\vee } = \bigoplus _{i \in I} \mathbb {Z} \alpha _i^{\vee }$ the coroot lattice of G, one has a semi-infinite Schubert variety $\mathbf {Q}_{G}(x)\subset \mathbf {Q}_{G}^{\mathrm {rat}}$ , which is infinite-dimensional and is given as an orbit closure for the Iwahori subgroup $\mathbf {I}\subset G(\mathbb {C}[\![z]\!])$ . We distinguish the semi-infinite Schubert variety $\mathbf {Q}_{G} := \mathbf {Q}_{G}(e) \subset \mathbf {Q}_{G}^{\mathrm {rat}}$ associated to the identity element e of the affine Weyl group, and also call it the semi-infinite flag manifold.

Our main object of study is the equivariant K-group $K_{H \times \mathbb {C}^{\ast }}(\mathbf {Q}_{G})$ –and that of $\mathbf {Q}_{G}^{\mathrm {rat}}$ , denoted by $K_{H \times \mathbb {C}^{\ast }}\big(\mathbf {Q}_{G}^{\mathrm {rat}}\big )$ –which is a variant of the K-group $K_{\tilde {\mathbf {I}}}^{\prime }(\mathbf {Q}_{G})$ introduced recently in [Reference Kato, Naito and Sagaki13]. Our K-group is a module over the equivariant scalar ring $\mathbb {Z}\left [q^{\pm 1}\right ][P]$ , where P is the weight lattice of G, $\mathbb {Z}[P]=\mathbb {Z}[e^{\mu } : \mu \in P]$ is the character ring of H and $q\in R(\mathbb {C}^*)$ is the character of loop rotation. Therefore, the K-group $K_{H \times \mathbb {C}^{\ast }}(\mathbf {Q}_{G})$ is a $\mathbb {Z}\left [q^{\pm 1}\right ][P]$ -submodule of an extension of scalars of the equivariant K-group $K_{\tilde {\mathbf {I}}}^{\prime }(\mathbf {Q}_{G})$ of [Reference Kato, Naito and Sagaki13] with respect to the Iwahori subgroup $\mathbf {I}$ and loop rotation.

A fundamental result of [Reference Kato, Naito and Sagaki13] is the combinatorial Chevalley formula for dominant weights in the K-group $K_{\tilde {\mathbf {I}}}^{\prime }(\mathbf {Q}_{G})$ (and hence in $K_{H \times \mathbb {C}^{\ast }}(\mathbf {Q}_{G})$ ). This formula describes, in terms of semi-infinite Lakshmibai–Seshadri paths, the tensor product of the class of the line bundle $\left [\mathcal {O}_{\mathbf {Q}_{G}}(\lambda )\right ]$ associated to a dominant weight $\lambda \in P^+$ with the class of the structure sheaf $\left [\mathcal {O}_{\mathbf {Q}_{G}(x)}\right ]$ of a semi-infinite Schubert variety $\mathbf {Q}_{G}(x) \subset \mathbf {Q}_{G}$ for $x = w t_{\xi } \in W_{\mathrm {af}}^{\geq 0} := W \times Q^{\vee ,+} \subset W_{\mathrm {af}}$ , where $Q^{\vee ,+} := \sum _{i \in I} \mathbb {Z}_{\geq 0} \alpha _{i}^{\vee } \subset Q^{\vee }$ . This was followed up in [Reference Naito, Orr and Sagaki24] by another combinatorial Chevalley formula in $K_{H\times \mathbb {C}^*}(\mathbf {Q}_{G})$ , giving the tensor product of a Schubert class with an antidominant line bundle. The two Chevalley formulas–dominant [Reference Kato, Naito and Sagaki13] and antidominant [Reference Naito, Orr and Sagaki24]–were unified in [Reference Lenart, Naito and Sagaki19], giving the general Chevalley formula in $K_{H\times \mathbb {C}^*}(\mathbf {Q}_{G})$ for arbitrary weights $\lambda \in P$ .

The Chevalley formulas of [Reference Kato, Naito and Sagaki13Reference Naito, Orr and Sagaki24Reference Lenart, Naito and Sagaki19] thus provide the complete analogue for semi-infinite flag manifolds of their previously well-understood K-theory counterparts for the standard Kac–Moody flag varieties [Reference Pittie and Ram26Reference Littelmann and Seshadri22Reference Griffeth and Ram7Reference Lenart and Postnikov20Reference Lenart and Shimozono21]. In all such formulas, the objective is to expand the tensor product of a Schubert class with an equivariant line bundle, as a linear combination of Schubert classes with equivariant scalar coefficients. In the case of $\mathbf {Q}_{G}$ , by [Reference Lenart, Naito and Sagaki19], this takes the form

(1.1) $$ \begin{align} \left[\mathcal{O}_{\mathbf{Q}_{G}(x)}(\lambda)\right] = \sum_{\substack{y\in W_{\mathrm{af}}^{\ge 0}\\\mu\in P}} c_{x,y}^{\lambda,\mu} \cdot e^\mu \cdot \left[\mathcal{O}_{\mathbf{Q}_{G}(y)}\right], \end{align} $$

with $x,y\in W_{\mathrm {af}}^{\ge 0}$ , $\lambda ,\mu \in P$ and $c_{x,y}^{\lambda ,\mu }\in \mathbb {Z}\left [q^{\pm 1}\right ]$ . We note that the sum on the right-hand side of equation (1.1), while generally infinite, satisfies a notion of convergence introduced in [Reference Kato, Naito and Sagaki13].

In this paper, we shall study the inverseFootnote 1 expansion in $K_{H\times \mathbb {C}^*}(\mathbf {Q}_{G})$ :

(1.2) $$ \begin{align} e^\lambda\cdot \left[\mathcal{O}_{\mathbf{Q}_{G}(x)}\right] &= \sum_{\substack{y\in W_{\mathrm{af}}^{\ge 0}\\\mu\in P}} d_{x,y}^{\lambda,\mu}\cdot \left[\mathcal{O}_{\mathbf{Q}_{G}(y)}(\mu)\right] \end{align} $$

for $x\in W_{\mathrm {af}}^{\ge 0}$ and $\lambda \in P$ . In contrast to equation (1.1), the expansion in equation (1.2) is finite, as established in [Reference Orr25] for simply laced G–namely, $d_{x,y}^{\lambda ,\mu }\in \mathbb {Z}\left [q^{\pm 1}\right ]$ for any $\lambda ,\mu \in P$ and $x,y\in W_{\mathrm {af}}^{\ge 0}$ , and the right-hand side of equation (1.2) is always a finite sum. (These properties are expected to hold for arbitrary G.)

We call any formula for the right-hand side of equation (1.2) an inverse Chevalley formula for $\lambda \in P$ in $K_{H\times \mathbb {C}^*}(\mathbf {Q}_{G})$ . The analogous expansion for finite-dimensional flag manifolds $G/B$ was studied by Mathieu [Reference Mathieu23] in the context of filtrations of B-modules (compare [Reference Mathieu23, p. 239]). In fact, by [Reference Kato10, Proof of Proposition 1.15], one has a surjective map of $\mathbb {Z}[P]$ -modules given by

(1.3) $$ \begin{align} K_{H\times\mathbb{C}^*}(\mathbf{Q}_{G}) &\overset{\mathrm{trunc}}{\longrightarrow} K_H(G/B)\notag\\ q &\longmapsto 1\notag\\ \left[\mathcal{O}_{\mathbf{Q}_{G}(y)}\right] &\longmapsto \begin{cases}\left[\mathcal{O}_{X(y)}\right] &\text{if}\ y\in W,\\0 &\text{otherwise,}\end{cases} \end{align} $$

where $\left [\mathcal {O}_{X(y)}\right ]$ for $y\in W$ denotes the structure sheaf of the Schubert variety

(1.4) $$ \begin{align} X(y)=\overline{Byw_{\circ} B/B}\subset G/B \end{align} $$

and $w_{\circ }$ is the longest element of W. The map $\mathrm {trunc}$ also respects the tensor product by equivariant line bundles. Thus, for $x\in W$ , the image of equation (1.2) under $\mathrm {trunc}$ produces the analogous inverse Chevalley expansion in $K_H(G/B)$ (see also Remark 3.13). In this sense, any inverse Chevalley formula (1.2) must incorporate the necessary ‘corrections’ in $K_{H\times \mathbb {C}^*}(\mathbf {Q}_{G})$ to the classical inverse Chevalley formulas in $K_H(G/B)$ .

We note that in the case of $K_H(G/B)$ , there is a simple transformation to pass between ordinary and inverse Chevalley formulas (as explained in, e.g., [Reference Mathieu23, p. 239]). The lack of such a transformation for $K_{H\times \mathbb {C}^*}(\mathbf {Q}_{G})$ justifies the independent study of inverse Chevalley formulas in the semi-infinite setting. Moreover, the finiteness of equation (1.2) offers an advantage over equation (1.1).

The purpose of this paper is to prove a completely explicit, combinatorial inverse Chevalley formula in the equivariant K-group $K_{H \times \mathbb {C}^*}(\mathbf {Q}_{G})$ in the case of a simply laced group G and a minuscule weight $\lambda \in P$ . Before stating our results more precisely, let us discuss further motivations for this work.

1.1. Nil-DAHA and q-Heisenberg actions on $K_{H \times \mathbb {C}^*}\big(\mathbf {Q}_{G}^{\mathrm {rat}}\big )$

One approach to understanding equations (1.1) and (1.2) is that they relate two actions of the group algebra $\mathbb {Z}[P]$ on $K_{H \times \mathbb {C}^*}\big(\mathbf {Q}_{G}^{\mathrm {rat}}\big )$ , one given by equivariant scalar multiplication (the left-hand side of equation (1.2)) and the other by the tensor product with equivariant line bundles (the left-hand side of equation (1.1)). These actions of $\mathbb {Z}[P]$ extend to that of two distinct algebras on $K_{H \times \mathbb {C}^*}\big(\mathbf {Q}_{G}^{\mathrm {rat}}\big )$ : the nil double affine Hecke algebra (nil-DAHA) and a q-Heisenberg algebra.

Multiplication by equivariant scalars extends to a left action of the nil-DAHA $\mathbb {H}_0$ on $K_{H \times \mathbb {C}^*}\big(\mathbf {Q}_{G}^{\mathrm {rat}}\big )$ , in a way that is conceptually similar to the action of nil-Hecke algebras on the equivariant K-theory of Kac–Moody flag varieties [Reference Kostant and Kumar8]. On the right, however, instead of a nil-Hecke algebra, one has an action of a q-Heisenberg algebra $\mathfrak {H}$ . This is generated by tensor products with equivariant line bundles $[\mathcal {O}(\lambda )]$ ( $\lambda \in P$ ) and translations $Q^\vee \cong H(\mathbb {C}\left (\!\left (z\right )\!\right ))/H(\mathbb {C}[\![z]\!])$ . The two actions commute, making $K_{H \times \mathbb {C}^*}\big(\mathbf {Q}_{G}^{\mathrm {rat}}\big )$ an $(\mathbb {H}_0,\mathfrak {H})$ -bimodule.

This bimodule structure is a fundamental tool in the study of $K_{H \times \mathbb {C}^*}\big(\mathbf {Q}_{G}^{\mathrm {rat}}\big )$ , as standard methods such as localisation are not available. Furthermore, as explained in [Reference Orr25], it gives a geometric realisation of the nonsymmetric q-Toda system introduced in [Reference Cherednik and Orr4] (see [Reference Braverman and Finkelberg1Reference Braverman and Finkelberg2Reference Givental and Lee5] for related results on the usual q-Toda system and [Reference Koroteev14Reference Koroteev and Zeitlin15] for its $(\mathsf {q},\mathsf {t})$ -extension given by Macdonald difference operators in type A). For us, the fact that the $\mathbb {H}_0$ -action on $K_{H \times \mathbb {C}^*}\big(\mathbf {Q}_{G}^{\mathrm {rat}}\big )$ includes the operators of multiplication by equivariant scalars is key. We use the limit construction [Reference Orr25] of the $\mathbb {H}_0$ -action on $K_{H \times \mathbb {C}^*}\big(\mathbf {Q}_{G}^{\mathrm {rat}}\big )$ to find and prove our inverse Chevalley formula in $K_{H\times \mathbb {C}^*}(\mathbf {Q}_{G})$ , which is given by Theorem 1.

1.2. Quantum K-theory of $G/B$

In [Reference Kato10Reference Kato12], Kato established a $\mathbb {Z}[P]$ -module isomorphism – with $\mathbb {Z}[P]$ acting by equivariant scalars – from the (completed) H-equivariant quantum K-group $QK_{H}(G/B) := K_{H}(G/B) \otimes \mathbb {Z}\left [\!\left [Q^{\vee ,+}\right ]\!\right ]$ of the finite-dimensional flag manifold $G/B$ onto the H-equivariant K-group $K_{H}(\mathbf {Q}_{G})$ obtained by specialising $q=1$ in $K_{H\times \mathbb {C}^*}(\mathbf {Q}_{G})$ . (Here $\mathbb {Z}\left [\!\left [Q^{\vee ,+}\right ]\!\right ]$ is the ring of formal power series in the (Novikov) variables $Q_{i}$ , $i \in I$ .) Kato’s isomorphism respects Schubert classes and intertwines the quantum multiplication in $QK_{H}(G/B)$ with the tensor product by line bundles in $K_H(\mathbf {Q}_{G})$ . Thus it provides a means to transport formulas from $K_H(\mathbf {Q}_{G})$ to $QK_H(G/B)$ . In the sequel [Reference Kouno, Naito, Orr and Sagaki16] to this paper, we will use Kato’s isomorphism to derive a corresponding inverse Chevalley formula in $QK_{H}(G/B)$ .

1.3. Our results

Let us now explain our results in more detail. Recall that a weight $\lambda \in P$ is called minuscule if $\left \langle \lambda ,\alpha ^\vee \right \rangle \in \{0,\pm 1\}$ for all $\alpha \in \Delta $ . Nonzero minuscule weights exist in all types except $E_8$ , $F_4$ and $G_2$ .

Our results explicitly describe the inverse Chevalley formula in $K_{H\times \mathbb {C}^*}(\mathbf {Q}_{G})$ for arbitrary minuscule weights $\lambda \in P$ in the case when G is simply laced. By iteration, our formulas completely determine the inverse Chevalley rule for arbitrary weights in ADE type (except in type $E_8$ ). Indeed, a result of Stembridge [Reference Stembridge27] (as stated in [Reference Lenart18, Theorem 2.1]) asserts that in all types except $E_8$ , $F_4$ and $G_2$ , the minuscule weights form a set of generators for the weight lattice. (The full version of Stembridge’s result, which holds in arbitrary type, requires quasi-minuscule weights. We plan to take up the study of our constructions in the case of quasi-minuscule weights elsewhere.)

Any minuscule weight belongs to the Weyl group orbit of a dominant minuscule weight – that is, a minuscule fundamental weight. Suppose $\varpi _k\in P^+$ is a minuscule fundamental weight. Set $J=I\setminus \{k\}$ , and consider the parabolic subgroup $W_J=\left \langle s_j \mid j\in J\right \rangle $ , which is the stabiliser of $\varpi _k$ . Let $W^J$ be the set of minimal coset representatives for $W/W_J$ . Finally, let $\lambda =x\varpi _k\in P$ be an arbitrary minuscule weight, where $x\in W^J$ .

1.3.1. Algebraic formula

For G simply laced, $\lambda $ as before and any $w\in W$ , our first main result gives an algebraic expression for the product $e^\lambda \cdot \left [\mathcal {O}_{\mathbf {Q}_{G}(w)}\right ]\in K_{H\times \mathbb {C}^*}(\mathbf {Q}_{G})$ in terms of the right q-Heisenberg action on $K_{H\times \mathbb {C}^*}\big(\mathbf {Q}_{G}^{\mathrm {rat}}\big )$ . This is given explicitly as a sum over a set $\mathbf {QW}_{\lambda ,w}$ of walks $(w_1,\dotsc ,w_n)$ in the quantum Bruhat graph $\mathrm {QBG}(W)$ [Reference Brenti, Fomin and Postnikov3], beginning at $w_0=w$ and with steps prescribed by a set of positive roots determined by $\lambda $ . (Here n is the length of the minimal representative of the coset $w_{\circ } W_J$ .)

Theorem 1 (= Theorem 3.11)

Assume that G is of $ADE$ type but not of type $E_8$ . For any minuscule weight $\lambda =x\varpi _k\in P$ , where $x\in W^J$ , and any $w\in W$ , we have

(1.5) $$ \begin{align} e^\lambda\cdot\left[\mathcal{O}_{\mathbf{Q}_{G}(w)}\right] = \sum_{\mathbf{w}=(w_1,\dotsc,w_n)\in\mathbf{QW}_{\lambda,w}}\left[\mathcal{O}_{\mathbf{Q}_{G}(w_n)}\right]\cdot \widetilde{g}^+_{\mathbf{w}} X^{-w_{\circ} w_l^{-1}\lambda} \widetilde{g}^-_{\mathbf{w}}, \end{align} $$

where $l=\ell (x)$ and $\widetilde {g}^+_{\mathbf {w}} X^{-w_{\circ } w_l^{-1}\nu } \widetilde {g}^-_{\mathbf {w}}$ is an element of the q-Heisenberg algebra $\mathfrak {H}$ given explicitly by equations (3.63) and (3.64).

Acting by translations from the q-Heisenberg algebra, one immediately obtains a corresponding formula for $e^\lambda \cdot \left [\mathcal {O}_{\mathbf {Q}_{G}\left (wt_\xi \right )}\right ]\in K_{H\times \mathbb {C}^*}\big(\mathbf {Q}_{G}^{\mathrm {rat}}\big )$ , for any $\xi \in Q^\vee $ .

In order to prove Theorem 1, we apply the main result of [Reference Orr25]. The main step in our proof (see Theorem 3.7) is the intricate computation of a limit, as $\mathsf {t}\to 0$ , from the polynomial representation of the double affine Hecke algebra (DAHA). Thus one should also regard Theorem 1 as an explicit determination of all nonsymmetric q-Toda operators, in the sense of [Reference Cherednik and Orr4Reference Orr25], for minuscule weights in ADE type.

1.3.2. Combinatorial formula

Our second main result expresses the same product $e^\lambda \cdot \left [\mathcal {O}_{\mathbf {Q}_{G}(w)}\right ]$ combinatorially. To achieve this, we enhance the set of quantum walks $\mathbf {QW}_{\lambda ,w}$ to a set $\widetilde {\mathbf {QW}}_{\lambda ,w}$ of decorated quantum walks. Roughly speaking, a decorated quantum walk $(\mathbf {w},\mathbf {b})\in \widetilde {\mathbf {QW}}_{\lambda ,w}$ consists of a quantum walk $\mathbf {w}=(w_1,\dotsc ,w_n)\in \mathbf {QW}_{\lambda ,w}$ together with a decoration $\mathbf {b}: S(\mathbf {w})\to \{0,1\}$ . The latter is a $\{0,1\}$ -valued function on an explicit subset $S(\mathbf {w})\subset \{t : w_t=w_{t-1}\}$ of the stationary steps in the walk $\mathbf {w}$ . Each $(\mathbf {w},\mathbf {b})\in \widetilde {\mathbf {QW}}_{\lambda ,w}$ carries a sign $(-1)^{(\mathbf {w},\mathbf {b})}\in \{\pm 1\}$ , a weight $\mathrm {wt}(\mathbf {w},\mathbf {b})\in Q^\vee $ and a degree $\deg (\mathbf {w},\mathbf {b})\in \mathbb {Z}$ . For details, see Section 3.4.1.

Our combinatorial inverse Chevalley formula reads as follows:

Theorem 2 (= Theorem 3.14)

Assume that G is of $ADE$ type but not of type $E_8$ . For any minuscule weight $\lambda =x\varpi _k\in P$ , where $x\in W^J$ , and any $w\in W$ , we have

(1.6) $$ \begin{align} &e^{\lambda}\cdot \left[\mathcal{O}_{\mathbf{Q}_{G}(w)}\right]\nonumber\\ &= \sum_{(\mathbf{w},\mathbf{b})\in\widetilde{\mathbf{QW}}_{\lambda,w}}(-1)^{(\mathbf{w},\mathbf{b})}q^{\deg(\mathbf{w},\mathbf{b})}\cdot[\mathcal{O}_{\mathbf{Q}_{G}(w_nt_{-w_\circ(\mathrm{wt}(\mathbf{w},\mathbf{b}))})}(-w_\circ w_l^{-1}\lambda+\mathrm{wt}(\mathbf{w},\mathbf{b}))]. \end{align} $$

Theorem 2 is obtained as an immediate consequence of Theorem 1, by fully expanding the right-hand side of equation (1.5) in the q-Heisenberg algebra. Our combinatorial framework is designed to record the terms in this expansion. We find it satisfying that the DAHA-based limit used to prove equation (1.5) automatically manufactures the combinatorics of decorated quantum walks necessary for both formulas. We mention that equation (1.5) gives the extension to $K_{H\times \mathbb {C}^*}(\mathbf {Q}_{G})$ of Lenart’s rule [Reference Lenart17, Theorem 3.1], which holds in $K(SL(n+1)/B)$ , for multiplying a Grothendieck polynomial by a variable.

In Appendix A we work out some further details in the type A case, including concrete examples of equation (1.5) in equations (A.1) and (A.2) and the important special case $w=w_\circ $ , $k=1$ in equation (A.7). We also explain, following [Reference Orr25, §5.1], how to obtain the usual q-Toda difference operator by symmetrising Theorem 1.

1.4. The sequel

In [Reference Kouno, Naito, Orr and Sagaki16] we will establish a different, equivalent version of our inverse Chevalley formula (1.6) in terms of paths (instead of walks) in the quantum Bruhat graph. By means of this alternate formula, we will give a separate and logically independent proof of Theorem 2 in type A, based on the Chevalley formulas of [Reference Kato, Naito and Sagaki13Reference Naito, Orr and Sagaki24] and an equivalent set of character identities for Demazure submodules of level $0$ extremal weight modules. Finally, we will use Theorem 2 to derive a corresponding inverse Chevalley formula in the quantum K-ring $QK_H(G/B)$ , by means of Kato’s isomorphism.

2. Basic notation

2.1. Root system

Let G be a simply connected simple algebraic group over $\mathbb {C}$ . As in the introduction, we fix a maximal torus and Borel subgroup $H\subset B\subset G$ . Set $\mathfrak {g} := \mathrm {Lie}(G)$ and $\mathfrak {h} := \mathrm {Lie}(H)$ . We denote by $\left \langle \cdot , \cdot \right \rangle : \mathfrak {h}^{\ast } \times \mathfrak {h} \rightarrow \mathbb {C}$ the canonical pairing, where $\mathfrak {h}^{\ast } = \mathrm {Hom}_{\mathbb {C}}(\mathfrak {h}, \mathbb {C})$ .

Let $\Delta \subset \mathfrak {h}^{\ast }$ be the root system of $\mathfrak {g}$ , $\Delta ^{+} \subset \Delta $ the positive roots (with respect to B) and $\{ \alpha _{i} \}_{i \in I} \subset \Delta ^{+}$ the set of simple roots. We denote by $\alpha ^{\vee } \in \mathfrak {h}$ the coroot corresponding to $\alpha \in \Delta $ . Also, we denote by $\theta \in \Delta ^+$ the highest root of $\Delta $ , and we set $\rho := (1/2) \sum _{\alpha \in \Delta ^{+}} \alpha $ . The root lattice Q and the coroot lattice $Q^{\vee }$ of $\mathfrak {g}$ are $Q := \sum _{i \in I} \mathbb {Z} \alpha _{i}$ and $Q^{\vee } := \sum _{i \in I} \mathbb {Z} \alpha _{i}^{\vee }$ .

For $i \in I$ , let $\varpi _{i} \in \mathfrak {h}^{\ast }$ be the fundamental weight determined by $\left \langle \varpi _{i}, \alpha _{j}^{\vee } \right \rangle = \delta _{i, j}$ for all $j \in I$ , where $\delta _{i, j}$ denotes the Kronecker delta. The weight lattice P of $\mathfrak {g}$ is defined by $P := \sum _{i \in I} \mathbb {Z} \varpi _{i}$ . We denote by $\mathbb {Z}[P]$ the group algebra of P – that is, the associative algebra generated by formal elements $\left \{ e^{\lambda } \mid \lambda \in P\right \}$ , where the product is defined by $e^{\lambda } e^{\mu } := e^{\lambda + \mu }$ for $\lambda , \mu \in P$ .

A reflection $s_{\alpha } \in GL(\mathfrak {h}^{\ast })$ , $\alpha \in \Delta $ , is defined by $s_{\alpha }(\lambda ) := \lambda - \left \langle \lambda , \alpha ^{\vee } \right \rangle \alpha $ for $\lambda \in \mathfrak {h}^{\ast }$ . We write $s_{i} := s_{\alpha _{i}}$ for $i \in I$ . Then the Weyl group $W := \langle s_{i} \mid i \in I \rangle $ of $\mathfrak {g}$ is the subgroup of $GL(\mathfrak {h}^{\ast })$ generated by $\{ s_{i} \}_{i \in I}$ . We denote by $\ell (w)$ the length of $w\in W$ with respect to $\{ s_{i} \}_{i \in I}$ .

2.2. Quantum Bruhat graph

The quantum Bruhat graph $\mathrm {QBG}(W)$ (compare [Reference Brenti, Fomin and Postnikov3, Definition 6.1]) is the $\Delta ^+$ -labelled directed graph whose vertices are the elements of W and whose edges are of the form $x \xrightarrow {\alpha } y$ , with $x, y \in W$ and $\alpha \in \Delta ^+$ , such that $y = x s_{\alpha }$ and either of the following holds: (B) $\ell (y) = \ell (x) + 1$ or (Q) $\ell (y) = \ell (x) - 2\left \langle \rho , \alpha ^\vee \right \rangle + 1$ . An edge satisfying (B) (resp., (Q)) is called a Bruhat edge (resp., a quantum edge).

2.3. Affine root system

Let $\mathfrak {g}_{\mathrm {af}} := \left (\mathfrak {g} \otimes \mathbb {C}\left [z, z^{-1}\right ]\right ) \oplus \mathbb {C} c \oplus \mathbb {C} d$ be the (untwisted) affine Lie algebra over $\mathbb {C}$ associated to $\mathfrak {g}$ , where c is the canonical central element and d is the degree operator. Then $\mathfrak {h}_{\mathrm {af}} := \mathfrak {h} \oplus \mathbb {C} c \oplus \mathbb {C} d$ is the Cartan subalgebra of $\mathfrak {g}_{\mathrm {af}}$ . We denote by $\left \langle \cdot , \cdot \right \rangle : \mathfrak {h}_{\mathrm {af}}^{\ast } \times \mathfrak {h}_{\mathrm {af}} \rightarrow \mathbb {C}$ the canonical pairing. Regarding $\lambda \in \mathfrak {h}^{\ast }$ as $\lambda \in \mathfrak {h}_{\mathrm {af}}^{\ast } = \mathrm {Hom}_{\mathbb {C}}(\mathfrak {h}_{\mathrm {af}}, \mathbb {C})$ by setting $\left \langle \lambda , c \right \rangle = \left \langle \lambda , d \right \rangle = 0$ , we have $\mathfrak {h}^{\ast } \subset \mathfrak {h}_{\mathrm {af}}^{\ast }$ . In this identification, we see that the canonical pairing $\left \langle \cdot , \cdot \right \rangle $ on $\mathfrak {h}_{\mathrm {af}}^{\ast } \times \mathfrak {h}_{\mathrm {af}}$ extends that on $\mathfrak {h}^{\ast } \times \mathfrak {h}$ .

Let us consider the root system of $\mathfrak {g}_{\mathrm {af}}$ . We define $\delta $ to be the unique element of $\mathfrak {h}_{\mathrm {af}}^{\ast }$ which satisfies $\left \langle \delta , h \right \rangle = 0$ for all $h \in \mathfrak {h}$ , $\left \langle \delta , c \right \rangle = 0$ and $\left \langle \delta , d \right \rangle = 1$ . We set $\alpha _{0} := -\theta + \delta \in \mathfrak {h}_{\mathrm {af}}^{\ast }$ . Then the root system $\Delta _{\mathrm {af}}$ of $\mathfrak {g}_{\mathrm {af}}$ has simple roots $\{ \alpha _{i} \}_{i \in I_{\mathrm {af}}}$ , where $I_{\mathrm {af}} := I \sqcup \{ 0 \}$ .

For each $\alpha \in \Delta _{\mathrm {af}}$ , we have a reflection $s_{\alpha } \in GL(\mathfrak {h}_{\mathrm {af}})$ , defined as for $\mathfrak {g}$ . Note that for $\alpha \in \Delta \subset \Delta _{\mathrm {af}}$ , the restriction of a reflection $s_{\alpha }$ defined on $\mathfrak {h}_{\mathrm {af}}$ to $\mathfrak {h}$ coincides with a reflection $s_{\alpha }$ defined on $\mathfrak {h}$ . Set $s_{i} := s_{\alpha _{i}}$ for $i \in I_{\mathrm {af}}$ . Then the Weyl group $W_{\mathrm {af}}$ of $\mathfrak {g}_{\mathrm {af}}$ (called the affine Weyl group) is defined to be the subgroup of $GL(\mathfrak {h}_{\mathrm {af}})$ generated by $\{ s_{i} \}_{i \in I_{\mathrm {af}}}$ , namely $W_{\mathrm {af}} = \langle s_{i} \mid i \in I_{\mathrm {af}} \rangle $ . In [Reference Kac9, §6.5], it is shown that $W_{\mathrm {af}} \simeq W \ltimes \left \{ t_{\alpha ^{\vee }} \mid \alpha ^{\vee } \in Q^{\vee } \right \} \simeq W \ltimes Q^{\vee }$ , where $t_{\alpha ^{\vee }}$ is the translation element corresponding to $\alpha ^{\vee } \in Q^{\vee }$ .

2.4. Simply laced assumption

While the definitions we have given are completely general, we will in fact assume throughout that G is simply laced. As a result, we almost always identify $\mathfrak {h}^\ast $ with $\mathfrak {h}$ , roots with coroots and so on. We do this by means of the nondegenerate W-invariant symmetric bilinear form $(\cdot ,\cdot ) : \mathfrak {h}^* \times \mathfrak {h}^* \to \mathbb {C}$ , normalised so that $(\alpha ,\alpha )=2$ for all $\alpha \in \Delta $ . We write $\lvert \lambda \rvert ^2=(\lambda ,\lambda )$ for $\lambda \in \mathfrak {h}^*$ .

2.5. Extended affine Weyl group

Let $W_{\mathrm {ex}}=W\ltimes P$ be the extended affine Weyl group, which we let act on $P\oplus \mathbb {Z}\frac {\delta }{e}$ by the level $0$ action

(2.1) $$ \begin{align} wt_\mu (\lambda)=w(\lambda)-(\mu,\lambda)\delta,\qquad wt_\mu(\delta)=\delta. \end{align} $$

Here we choose e to be the smallest positive integer such that $e\cdot (P,P)\subset \mathbb {Z}$ .

We also define the group $\Pi =P/Q$ , which we realise as the subgroup of length $0$ elements in $W_{\mathrm {ex}}$ .

2.6. Parameters

Let us introduce a parameter $q^{1/e}$ such that $\left (q^{1/e}\right )^e=q$ , where $q\in R(\mathbb {C}^*)$ is the equivariant parameter corresponding to loop rotation on $\mathbf {Q}_{G}^{\mathrm {rat}}$ . We will also use a related, but distinct parameter $\mathsf {q}^{1/e}$ , as well as parameters $\mathsf {t},\mathsf {v}$ such that $\mathsf {t}=\mathsf {v}^2$ . Our base field for DAHA constructions will be $\mathsf {K}=\mathbb {Q}\left (\mathsf {q}^{1/e},\mathsf {v}\right )$ .

2.7. Matrices

For any ring R with $1$ , let $\mathrm {Mat}_W(R)$ denote the R-algebra of $W\times W$ matrices with entries in R. Let $\{e_w\}_{w\in W}$ be the standard basis of the free R-module $R^{\lvert W\rvert }$ . For $A\in \mathrm {Mat}_W(R)$ , we denote by $A_{w,\bullet }=\sum _{v\in W}A_{w,v}e_v$ the row of the matrix A indexed by $w\in W$ .

3. Inverse Chevalley formula via DAHA

In this section we use the methods of [Reference Orr25], based on the $(\mathbb {H}_0,\mathfrak {H})$ -bimodule structure of $K_{H \times \mathbb {C}^*}\big(\mathbf {Q}_{G}^{\mathrm {rat}}\big )$ , to prove the inverse Chevalley formulas (1.5) and (1.6).

3.1. K-groups

Let $K_{\tilde {\mathbf {I}}}\big(\mathbf {Q}_{G}^{\mathrm {rat}}\big )$ be the equivariant K-group of $\mathbf {Q}_{G}^{\mathrm {rat}}$ introduced in [Reference Kato, Naito and Sagaki13, §6], where $\tilde {\mathbf {I}}=\mathbf {I}\rtimes \mathbb {C}^*$ is a semidirect product of the Iwahori subgroup $\mathbf {I}\subset G(\mathbb {C}[\![z]\!])$ and loop rotation $\mathbb {C}^*$ . Correspondingly, $K_{\tilde {\mathbf {I}}}\big(\mathbf {Q}_{G}^{\mathrm {rat}}\big )$ is a module over $\mathbb {Z}[P]\left (\!\left (q^{-1}\right )\!\right )$ , which acts by equivariant scalar multiplication.

One has the following classes in $K_{\tilde {\mathbf {I}}}\big(\mathbf {Q}_{G}^{\mathrm {rat}}\big )$ , for each $x\in W_{\mathrm {af}}$ and $\lambda \in P$ :

  • Schubert classes $\left [\mathcal {O}_{\mathbf {Q}_{G}(x)}\right ]$

  • Equivariant line bundle classes $[\mathcal {O}(\lambda )]$

  • Classes $\left [\mathcal {O}_{\mathbf {Q}_{G}(x)}(\lambda )\right ]$ corresponding to the tensor product sheaves $\mathcal {O}_{\mathbf {Q}_{G}(x)}\otimes \mathcal {O}(\lambda )$ .

We follow the conventions of [Reference Kato, Naito and Sagaki13] for indexing equivariant line bundles and Schubert varieties in $\mathbf {Q}_{G}^{\mathrm {rat}}$ .

Definition 3.1 ( $(H\times \mathbb {C}^*)$ -equivariant K-groups of $\mathbf {Q}_{G}^{\mathrm {rat}}$ and $\mathbf {Q}_{G}$ )

Let $K_{H\times \mathbb {C}^*}\big(\mathbf {Q}_{G}^{\mathrm {rat}}\big )$ be the $\mathbb {Z}\left [q^{\pm 1}\right ][P]$ -submodule of $K_{\tilde {\mathbf {I}}}\big(\mathbf {Q}_{G}^{\mathrm {rat}}\big )$ consisting of all convergent infinite $\mathbb {Z}\left [q^{\pm 1}\right ][P]$ -linear combinations of Schubert classes $\left [\mathcal {O}_{\mathbf {Q}_{G}(x)}\right ]$ for $x\in W_{\mathrm {af}}$ , where convergence holds in the sense of [Reference Kato, Naito and Sagaki13, Proposition 5.11].

Similarly, we define $K_{H\times \mathbb {C}^*}(\mathbf {Q}_{G})$ to be the $\mathbb {Z}\left [q^{\pm 1}\right ][P]$ -submodule of $K_{\tilde {\mathbf {I}}}\big(\mathbf {Q}_{G}^{\mathrm {rat}}\big )$ consisting of all convergent infinite $\mathbb {Z}\left [q^{\pm 1}\right ][P]$ -linear combinations of Schubert classes $\left [\mathcal {O}_{\mathbf {Q}_{G}(x)}\right ]$ for $x\in W_{\mathrm {af}}^{\ge 0}$ .

The classes $\left \{\left [\mathcal {O}_{\mathbf {Q}_{G}(x)}\right ]\right \}_{x\in W_{\mathrm {af}}}$ satisfy a notion of topological linear independence in $K_{\tilde {\mathbf {I}}}\big(\mathbf {Q}_{G}^{\mathrm {rat}}\big )$ given by [Reference Kato, Naito and Sagaki13, Proposition 5.11]; thus they form a topological $\mathbb {Z}\left [q^{\pm 1}\right ][P]$ -basis of $K_{H\times \mathbb {C}^*}\big(\mathbf {Q}_{G}^{\mathrm {rat}}\big )$ . Also, one has $\left [\mathcal {O}_{\mathbf {Q}_{G}(x)}(\lambda )\right ]\in K_{H\times \mathbb {C}^*}\big(\mathbf {Q}_{G}^{\mathrm {rat}}\big )$ for any $x\in W_{\mathrm {af}}$ and $\lambda \in P$ , thanks to the two Chevalley formulas for dominant weights [Reference Kato, Naito and Sagaki13] and antidominant weights [Reference Naito, Orr and Sagaki24]. Similar (in fact, equivalent) assertions hold for $K_{H\times \mathbb {C}^*}(\mathbf {Q}_{G})$ .

Definition 3.2. Define $\mathbb {K}\subset K_{H \times \mathbb {C}^*}\big(\mathbf {Q}_{G}^{\mathrm {rat}}\big )$ to be the $\mathbb {Z}\left [q^{\pm 1}\right ]$ -submodule consisting of all finite $\mathbb {Z}\left [q^{\pm 1}\right ]$ -linear combinations of the classes $\left \{\left [\mathcal {O}_{\mathbf {Q}_{G}(x)}(\lambda )\right ]\right \}_{x\in W_{\mathrm {af}},\lambda \in P}$ .

By definition, $\mathbb {K}$ is only a $\mathbb {Z}\left [q^{\pm 1}\right ]$ -submodule of $K_{H \times \mathbb {C}^*}\big(\mathbf {Q}_{G}^{\mathrm {rat}}\big )$ . But, as shown in [Reference Orr25, Theorem 5.1], it is indeed a $\mathbb {Z}\left [q^{\pm 1}\right ][P]$ -submodule of $K_{H \times \mathbb {C}^*}\big(\mathbf {Q}_{G}^{\mathrm {rat}}\big )$ . We note that the classes $\left \{\left [\mathcal {O}_{\mathbf {Q}_{G}(x)}(\lambda )\right ]\right \}_{x\in W_{\mathrm {af}},\lambda \in P}$ are linearly independent over $\mathbb {Z}\left [q^{\pm 1}\right ]$ , by the Chevalley formula of [Reference Kato, Naito and Sagaki13].

To summarise, we have the following chain of $\mathbb {Z}\left [q^{\pm 1}\right ][P]$ -modules, and $K_{\tilde {\mathbf {I}}}\big(\mathbf {Q}_{G}^{\mathrm {rat}}\big )$ is in fact a $\mathbb {Z}[P]\left (\!\left (q^{-1}\right )\!\right )$ -module:

$$ \begin{align*}\mathbb{K}\subset K_{H \times \mathbb{C}^*}\left(\mathbf{Q}_{G}^{\mathrm{rat}}\right)\subset K_{\tilde{\mathbf{I}}}\left(\mathbf{Q}_{G}^{\mathrm{rat}}\right),\end{align*} $$

where $\mathbb {K}$ is equipped with the $\mathbb {Z}\left [q^{\pm 1}\right ]$ -basis $\left \{\left [\mathcal {O}_{\mathbf {Q}_{G}(x)}(\lambda )\right ]\right \}_{x\in W_{\mathrm {af}},\lambda \in P}$ , and $K_{H \times \mathbb {C}^*}\big(\mathbf {Q}_{G}^{\mathrm {rat}}\big )$ with the topological $\mathbb {Z}\left [q^{\pm 1}\right ][P]$ -basis $\left \{\left [\mathcal {O}_{\mathbf {Q}_{G}(x)}\right ]\right \}_{x\in W_{\mathrm {af}}}$ . Next we discuss additional structures on these modules.

3.1.1. q-Heisenberg algebra

Let $\widehat {\mathfrak {H}}$ be the q-Heisenberg algebra defined as a $\mathbb {Z}\left [q^{\pm 1/e}\right ]$ -algebra by generators

(3.1) $$ \begin{align} X^\nu\ (\nu\in P),\quad t_\mu\ (\mu\in P), \end{align} $$

and relations

(3.2) $$ \begin{align} &X^\lambda X^\nu = X^{\lambda+\nu} \end{align} $$

(3.3) $$ \begin{align} &t_\mu t_\xi = t_{\mu+\xi} \end{align} $$

(3.4) $$ \begin{align} &X^0 = t_0 = 1 \end{align} $$

(3.5) $$ \begin{align} &X^\nu t_\mu = q^{\left(\mu,\nu\right)}t_\mu X^\nu \end{align} $$

for all $\lambda ,\mu ,\nu ,\xi \in P$ .

Let $\mathfrak {H}\subset \widehat {\mathfrak {H}}$ be the $\mathbb {Z}\left [q^{\pm 1}\right ]$ -subalgebra generated by $X^\nu $ ( $\nu \in P$ ) and $t_\beta $ ( $\beta \in Q$ ).

Proposition 3.3. There exists a unique right $\mathfrak {H}$ -module structure on $\mathbb {K}$ and $K_{H \times \mathbb {C}^*}\big(\mathbf {Q}_{G}^{\mathrm {rat}}\big )$ such that

(3.6) $$ \begin{align} \left[\mathcal{O}_{\mathbf{Q}_{G}(x)}(\lambda)\right]\cdot X^\nu &= \left[\mathcal{O}_{\mathbf{Q}_{G}(x)}(\lambda+\nu)\right] \end{align} $$

(3.7) $$ \begin{align} \left[\mathcal{O}_{\mathbf{Q}_{G}(x)}(\lambda)\right]\cdot t_\beta &= q^{\left(\beta,\lambda\right)}\cdot\left[\mathcal{O}_{\mathbf{Q}_{G}\left(xt_{-w_\circ\beta}\right)}(\lambda)\right] \end{align} $$

for all $x\in W_{\mathrm {af}}$ , $\lambda ,\nu \in P$ and $\beta \in Q$ . Moreover, $\mathbb {K}$ is free as an $\mathfrak {H}$ -module, with $\mathfrak {H}$ -basis given by $\left \{\left [\mathcal {O}_{\mathbf {Q}_{G}(w)}\right ]\right \}_{w\in W}$ .

Proof. See [Reference Kato10] and [Reference Orr25, Proposition 2.3]. We note that our conventions here differ slightly from those of [Reference Orr25], where the class $\left [\mathcal {O}_{\mathbf {Q}_{G}(x)}\right ]$ is denoted by $\left [\overline {\mathcal {O}}_{x}\right ]$ . This accounts for the twist by $-w_\circ $ in the action of $t_\beta $ .

3.1.2. Nil-DAHA

Next we turn to the nil-DAHA $\mathbb {H}_0$ , which is the $\mathbb {Z}\left [\mathsf {q}^{\pm 1}\right ]$ -algebra (note the notational distinction between $\mathsf {q}$ and q) defined by generators

(3.8) $$ \begin{align} T_i \ (i\in I_{\mathrm{af}}),\quad X^{\nu} \ (\nu\in P), \end{align} $$

and relations

(3.9) $$ \begin{align} &T_i T_j\dotsm = T_j T_i\dotsm \qquad\left(m_{ij}=\left\lvert s_is_j\right\rvert \text{factors on both sides}\right) \end{align} $$

(3.10) $$ \begin{align} &T_i(T_i+1) = 0 \end{align} $$

(3.11) $$ \begin{align} &X^0=1 \end{align} $$

(3.12) $$ \begin{align} &X^{\nu}X^{\mu}=X^{\nu+\mu} \end{align} $$

(3.13) $$ \begin{align} &T_i X^{\nu} = X^{s_i(\nu)}T_i-\frac{X^{\nu}-X^{s_i(\nu)}}{1-X^{\alpha_i}}, \end{align} $$

where it is understood that $X^\delta =\mathsf {q}$ . We also use $D_i=1+T_i$ ( $i\in I_{\mathrm {af}}$ ). These elements satisfy the braid relations as before and quadratic relations $D_i^2=D_i$ .

Proposition 3.4 [Reference Kato, Naito and Sagaki13, Theorem 6.5]. There is a left $\mathbb {H}_0$ -action on $K_{H \times \mathbb {C}^*}\big(\mathbf {Q}_{G}^{\mathrm {rat}}\big )$ uniquely determined by

(3.14) $$ \begin{align} \mathsf{q}\cdot \left[\mathcal{O}_{\mathbf{Q}_{G}\left(t_\alpha\right)}(\lambda)\right] &= q^{-1}\left[\mathcal{O}_{\mathbf{Q}_{G}\left(t_\alpha\right)}(\lambda)\right] \end{align} $$

(3.15) $$ \begin{align} X^{\nu}\cdot\left[\mathcal{O}_{\mathbf{Q}_{G}\left(t_\alpha\right)}(\lambda)\right] &= e^{-\nu}\left[\mathcal{O}_{\mathbf{Q}_{G}\left(t_\alpha\right)}(\lambda)\right] \end{align} $$

(3.16) $$ \begin{align} D_i\cdot \left[\mathcal{O}_{\mathbf{Q}_{G}\left(t_\alpha\right)}(\lambda)\right] &= \left[\mathcal{O}_{\mathbf{Q}_{G}\left(t_\alpha\right)}(\lambda)\right] \end{align} $$

(3.17) $$ \begin{align} D_0\cdot \left[\mathcal{O}_{\mathbf{Q}_{G}\left(t_\alpha\right)}(\lambda)\right] &= \left[\mathcal{O}_{\mathbf{Q}_{G}\left(s_0t_\alpha\right)}(\lambda)\right] \end{align} $$

for all $i\in I$ , $\alpha \in Q$ and $\nu ,\lambda \in P$ .

We warn the reader that $X^\nu $ ( $\nu \in P$ ) can be viewed both as an element of $\mathbb {H}_0$ and as an element of $\mathfrak {H}$ . Its left and right actions on $K_{H \times \mathbb {C}^*}\big(\mathbf {Q}_{G}^{\mathrm {rat}}\big )$ differ drastically. From the left, $X^\nu \in \mathbb {H}_0$ acts as equivariant scalar multiplication by $e^{-\nu }$ , whereas from the right, $X^\nu \in \mathfrak {H}$ acts as the line bundle twist by $\mathcal {O}(\nu )$ .

3.1.3. $\mathbb {K}$ as a bimodule

By [Reference Orr25, Theorem 5.1], the $\mathbb {H}_0$ -action on $K_{H \times \mathbb {C}^*}\big(\mathbf {Q}_{G}^{\mathrm {rat}}\big )$ leaves $\mathbb {K}$ stable. Thus $\mathbb {K}$ is an $(\mathbb {H}_0,\mathfrak {H})$ -bimodule. Since $\mathbb {K}$ is free as a right $\mathfrak {H}$ -module, the nil-DAHA action on $\mathbb {K}$ and hence on $K_{H \times \mathbb {C}^*}\big(\mathbf {Q}_{G}^{\mathrm {rat}}\big )$ is therefore characterised by the unique ring homomorphism $\varrho _0 : \mathbb {H}_0 \to \mathrm {Mat}_W(\mathfrak {H})$ satisfying

(3.18) $$ \begin{align} H\cdot \left[\mathcal{O}_{\mathbf{Q}_{G}(w)}\right] = \sum_{v\in W}\left[\mathcal{O}_{\mathbf{Q}_{G}(v)}\right]\cdot \varrho_0(H)_{vw} \end{align} $$

for all $H\in \mathbb {H}_0$ and $w\in W$ .

For us, the key point is that the matrices encoding the inverse Chevalley formula (1.2) are exactly $\varrho _0\left (X^{-\lambda }\right )$ for $\lambda \in P$ .

3.1.4. Further observations

For any $i\in I_{\mathrm {af}}$ and $x\in W_{\mathrm {af}}$ , we have

(3.19) $$ \begin{align} D_i\cdot \left[\mathcal{O}_{\mathbf{Q}_{G}(x)}\right] = \begin{cases} \left[\mathcal{O}_{\mathbf{Q}_{G}\left(s_ix\right)}\right] &\text{if}\ s_ix\prec x,\\ \left[\mathcal{O}_{\mathbf{Q}_{G}(x)}\right] &\text{if}\ s_ix\succ x,\\ \end{cases} \end{align} $$

where $\prec $ is the semi-infinite Bruhat order on $W_{\mathrm {af}}$ (see [Reference Kato, Naito and Sagaki13] or [Reference Orr25]). In the special case where $i\in I$ and $x=w\in W$ , we have

(3.20) $$ \begin{align} D_i\cdot\left[\mathcal{O}_{\mathbf{Q}_{G}(w)}\right]= \begin{cases} \left[\mathcal{O}_{\mathbf{Q}_{G}\left(s_iw\right)}\right] &\text{if}\ s_iw<w,\\ \left[\mathcal{O}_{\mathbf{Q}_{G}(w)}\right] &\text{if}\ s_iw>w, \end{cases} \end{align} $$

where $<$ is the (usual) Bruhat order on W.

It follows from equation (3.20) and Proposition 3.4 (see also [Reference Kato, Naito and Sagaki13, Proposition 6.3]) that the nil affine Hecke $\mathbb {Z}\left [\mathsf {q}^{\pm 1}\right ]$ -subalgebra $\mathcal {H}_0\subset \mathbb {H}_0$ generated by

$$ \begin{align*} D_i \ (i\in I),\quad X^{\nu} \ (\nu\in P), \end{align*} $$

leaves $K_{H\times \mathbb {C}^*}(\mathbf {Q}_{G})\subset K_{H \times \mathbb {C}^*}\big(\mathbf {Q}_{G}^{\mathrm {rat}}\big )$ stable. (Our inverse Chevalley formulas make this explicit.)

We also deduce from equation (3.20) that $\left [\mathcal {O}_{\mathbf {Q}_{G}\left (w_\circ \right )}\right ]$ generates $\mathbb {K}$ as an $(\mathcal {H}_0,\mathfrak {H})$ -bimodule (in fact, the action of $\mathfrak {H}$ and the $D_i$ for $i\in I$ on $\left [\mathcal {O}_{\mathbf {Q}_{G}\left (w_\circ \right )}\right ]$ are sufficient to generate all of $\mathbb {K}$ ).

3.2. DAHA

The main result of [Reference Orr25] gives an algebraic construction of the homomorphism $\varrho _0 : \mathbb {H}_0\to \mathrm {Mat}_W(\mathfrak {H})$ of equation (3.18), as a limit as $\mathsf {t}\to 0$ from the polynomial representation of DAHA. Let us recall the necessary details of this construction.

3.2.1. Definition of DAHA

Recall that $\mathsf {K}=\mathbb {Q}\left (\mathsf {q}^{1/e},\mathsf {v}\right )$ , where $\mathsf {t}=\mathsf {v}^2$ . The (extended) DAHA $\widehat {\mathbb {H}}$ is the $\mathsf {K}$ -algebra defined by generators

(3.21) $$ \begin{align} T_i\ (i\in I_{\mathrm{af}}),\quad X^\nu\ (\nu\in P),\quad \pi\ (\pi\in \Pi), \end{align} $$

and relations

(3.22) $$ \begin{align} &T_i T_j\dotsm = T_j T_i \dotsm \qquad \left(m_{ij}=\left\lvert s_is_j\right\rvert \text{factors on both sides}\right) \end{align} $$

(3.23) $$ \begin{align} &(T_i-\mathsf{t})(T_i+1) = 0 \end{align} $$

(3.24) $$ \begin{align} &X^\nu X^\mu=X^{\nu+\mu} \end{align} $$

(3.25) $$ \begin{align} &X^0=1 \end{align} $$

(3.26) $$ \begin{align} &T_i X^\nu = X^{s_i(\nu)} T_i + (\mathsf{t}-1)\left(1-X^{\alpha_i}\right)^{-1}\left(X^\nu-X^{s_i(\nu)}\right) \end{align} $$

(3.27) $$ \begin{align} &\pi T_i \pi^{-1} = T_j \qquad \left(\pi(\alpha_i)=\alpha_j\right) \end{align} $$

(3.28) $$ \begin{align} &\pi X^\nu \pi^{-1} = X^{\pi(\nu)} \end{align} $$

for all $i,j\in I_{\mathrm {af}}$ , $\nu ,\mu \in P$ and $\pi \in \Pi $ . Here it is understood that $X^{\delta /e}=\mathsf {q}^{1/e}$ .

One has well-defined elements $Y^\nu \in \widehat {\mathbb {H}}$ ( $\nu \in P$ ) given by

(3.29) $$ \begin{align} Y^\nu = \mathsf{v}^{-\sum_{k=1}^\ell \epsilon_k}\pi T_{i_1}^{\epsilon_1}\dotsm T_{i_\ell}^{\epsilon_\ell} \end{align} $$

for any reduced expression $t_\nu = \pi s_{i_1}\dotsm s_{i_\ell }\in P\rtimes W=W_{\mathrm {ex}}$ , where $i_k\in I_{\mathrm {af}}$ and $\pi \in \Pi $ , and where $\epsilon _k\in \{\pm 1\}$ are determined by

(3.30) $$ \begin{align} \epsilon_k = \begin{cases} +1 &\text{if}\ \pi s_{i_1}\dotsm s_{i_{k-1}}\left(\alpha_{i_k}\right)\in-\Delta^++\mathbb{Z}\delta,\\-1 &\text{if}\ \pi s_{i_1}\dotsm s_{i_{k-1}}\left(\alpha_{i_k}\right)\in\Delta^++\mathbb{Z}\delta.\end{cases} \end{align} $$

These elements satisfy

(3.31) $$ \begin{align} &Y^\nu Y^\mu=Y^{\nu+\mu} \end{align} $$

(3.32) $$ \begin{align} &Y^0=1 \end{align} $$

(3.33) $$ \begin{align} &T_i Y^\nu - Y^{s_i(\nu)} T_i = (\mathsf{t}-1)\left(1-Y^{-\alpha_i}\right)^{-1}\left(Y^\nu-Y^{s_i(\nu)}\right) \end{align} $$

for any $i\in I$ and $\nu ,\mu \in P$ . (To extend the latter relation to $i\in I_{\mathrm {af}}$ , see, e.g., [Reference Orr25, §3.2].)

3.2.2. Polynomial representation

Let $\mathsf {K}(P)$ be the field of fractions of $\mathsf {K}[P]=\mathsf {K}[X^\nu : \nu \in P]$ . The extended affine Weyl group $W_{\mathrm {ex}}$ acts on $\mathsf {K}[P]$ by automorphisms as follows: $t_\mu w(X^\nu )=q^{-\left (\mu ,w\nu \right )}X^{w\nu }$ . By a ‘difference-reflection operator’ we mean an element of the smash product

$$ \begin{align*} \mathsf{K}(P)\rtimes W_{\mathrm{ex}}\cong\mathsf{K}(P)\otimes_{\mathsf{K}} \mathsf{K}[W_{\mathrm{ex}}] \quad \text{(}\mathsf{K}\text{-linear isomorphism)} \end{align*} $$

acting from the left on $\mathsf {K}(P)$ by multiplication operators (first tensor factor) and the induced action of $W_{\mathrm {ex}}$ (second tensor factor). We note that $\mathsf {K}[P]\rtimes P$ , where $P\cong \left \{t_\mu : \mu \in P\right \}\subset W_{\mathrm {ex}}$ , is nothing but the q-Heisenberg algebra $\widehat {\mathfrak {H}}$ with scalars extended to $\mathsf {K}$ .

The polynomial representation $\varrho : \widehat {\mathbb {H}}\to \mathrm {End}(\mathsf {K}[P])$ is a faithful representation of $\widehat {\mathbb {H}}$ given as follows: $\varrho (X^\nu )$ is multiplication by $X^\nu $ , the elements $\pi \in \Pi \subset W_{\mathrm {ex}}$ act by the automorphisms already discussed and $T_i$ ( $i\in I_{\mathrm {af}}$ ) act by Demazure–Lusztig operators:

(3.34) $$ \begin{align} \varrho(T_i) &= \frac{\mathsf{t}X^{\alpha_i}-1}{X^{\alpha_i}-1}s_i+\frac{1-\mathsf{t}}{X^{\alpha_i}-1}. \end{align} $$

In general, elements of $\widehat {\mathbb {H}}$ act on $\mathsf {K}[P]$ by difference-reflection operators from $\mathsf {K}(P)\rtimes W_{\mathrm {ex}}$ ; therefore, they are represented uniquely as sums of $ft_\mu u = f\otimes t_\mu u$ , where $f\in \mathsf {K}(P)$ and $t_\mu u\in W_{\mathrm {ex}}$ . Obviously, not all elements of $\mathsf {K}(P)\rtimes W_{\mathrm {ex}}$ leave $\mathsf {K}[P]$ stable, but those from $\widehat {\mathbb {H}}$ do. As in equation (3.34), we will simultaneously regard $\varrho (H)$ for $H\in \widehat {\mathbb {H}}$ as an element of $\mathrm {End}(\mathsf {K}[P])$ and as a difference-reflection operator – that is, an element of $\mathsf {K}(P)\rtimes W_{\mathrm {ex}}$ .

3.2.3. Limiting procedure

Consider the homomorphism

$$ \begin{align*} \varkappa : \mathsf{K}(P)\rtimes W_{\mathrm{ex}} \to \mathrm{Mat}_W(\mathsf{K}(P)\rtimes P), \end{align*} $$

which for $ft_\mu u\in \mathsf {K}(P)\rtimes W_{\mathrm {ex}}$ and $v,w\in W$ is given by

(3.35) $$ \begin{align} \varkappa\left(f t_\mu u\right)_{vw}= \begin{cases} \left.\left(v^{-1}\cdot f\right)\right\rvert_{X^\nu\mapsto \mathsf{t}^{-\left(\rho,\nu\right)}X^\nu}\mathsf{t}^{\left(\rho,v^{-1}\mu\right)}t_{v^{-1}(\mu)} &\text{if}\ v=uw,\\ 0 &\text{otherwise.} \end{cases} \end{align} $$

By [Reference Orr25, Theorem 5.1], the sought-after matrices $\varrho _0\left (X^{-\lambda }\right )$ from equation (3.18) can be obtained by applying a simple automorphism to the result of the entry-wise limit

(3.36) $$ \begin{align} \varrho^{\prime}_0\left(Y^\lambda\right):=\lim_{\mathsf{v}\to 0} \varkappa\left(\varrho\left(Y^{\lambda}\right)\right)\in\mathrm{Mat}_W(\mathfrak{H}). \end{align} $$

The existence of this limit for any $\lambda \in P$ is ensured by [Reference Cherednik and Orr4, Theorem 4.4]. The significance of $Y^{\lambda }$ here is that it is the image of $X^{-\lambda }$ under the DAHA duality anti-involution; see [Reference Orr25, §3.2] for further details.

3.3. Computing limits

Our goal is thus to compute certain $\varrho ^{\prime }_0\left (Y^\lambda \right )$ given by formula (3.36). To do so, we will work with compositions of the auxiliary difference-reflection operators

(3.37) $$ \begin{align} \mathsf{G}_\eta^\pm &= \frac{\mathsf{t}^{\pm 1}-X^{\eta}}{1-X^{\eta}}+\frac{\mathsf{t}^{\pm 1}-1}{1-X^{-\eta}} s_\eta \end{align} $$

for $\eta \in \Delta ^+_{\mathrm {af}}$ . These are cousins of the Demazure–Lusztig operators $\varrho (T_i)$ :

$$ \begin{align*} \mathsf{G}^+_{\alpha_i} &= s_i\varrho(T_i), \qquad \mathsf{G}^-_{\alpha_i} = \left(\mathsf{G}^+_{\alpha_i}\right)^{-1}= \varrho\left(T_i^{-1}\right)s_i. \end{align*} $$

As we wish to take $\mathsf {v}\to 0$ , all rational functions in such operators will be expanded in $\mathbb {Z}\left [\mathsf {q}^{\pm 1/e}\right ][P]\left (\!\left (\mathsf {v}\right )\!\right )$ . In this context, we shall write $A\approx B$ to mean that A and B have the same lowest term – that is, that A and B have the same order with respect to $\mathsf {v}$ and that $A-B$ has strictly greater order than that of A and B.

Set $\eta \in \Delta ^+$ . By equation (3.35), we see that a matrix entry of $\varkappa \left (\mathsf {G}_\eta ^\pm \right )$ vanishes unless it is indexed by $(w,w)$ or $\left (w,s_\eta w\right )$ for some $w\in W$ . The nonvanishing entries are given as follows:

(3.38) $$ \begin{align} \varkappa\left(\mathsf{G}_\eta^+\right)_{w,w} &= \frac{\mathsf{t}-\mathsf{t}^{-\left(\rho,w^{-1}\eta\right)}X^{w^{-1}(\eta)}}{1-\mathsf{t}^{-\left(\rho,w^{-1}\eta\right)}X^{w^{-1}(\eta)}} \approx \begin{cases} 1 & \text{if}\ w^{-1}\eta>0,\\ \mathsf{t}\left(1-X^{w^{-1}\eta}\right) & \text{if}\ \left(\rho,w^{-1}\eta\right)=-1,\\ \mathsf{t} & \text{if}\ \left(\rho,w^{-1}\eta\right)<-1 \end{cases} \end{align} $$

(3.39) $$ \begin{align} \varkappa\left(\mathsf{G}_\eta^+\right)_{w,s_\eta w} &= \frac{\mathsf{t}-1}{1-\mathsf{t}^{\left(\rho,w^{-1}\eta\right)}X^{-w^{-1}\eta}} \approx \begin{cases} -1 &\text{if}\ w^{-1}\eta>0,\\ \mathsf{t}^{-\left(\rho,w^{-1}\eta\right)}X^{w^{-1}\eta} & \text{if}\ w^{-1}\eta<0 \end{cases} \end{align} $$

(3.40) $$ \begin{align} \varkappa\left(\mathsf{G}_\eta^-\right)_{w,w} &= \frac{\mathsf{t}^{-1}-\mathsf{t}^{-\left(\rho,w^{-1}\eta\right)}X^{w^{-1}(\eta)}}{1-\mathsf{t}^{-\left(\rho,w^{-1}\eta\right)}X^{w^{-1}(\eta)}} \approx \begin{cases} 1-X^{-w^{-1}\eta} & \text{if}\ \left(\rho,w^{-1}\eta\right)=1,\\ 1 & \text{if}\ \left(\rho,w^{-1}\eta\right)>1,\\ \mathsf{t}^{-1} & \text{if}\ w^{-1}\eta<0 \end{cases} \end{align} $$

(3.41) $$ \begin{align} \varkappa\left(\mathsf{G}_\eta^-\right)_{w,s_\eta w} &= \frac{\mathsf{t}^{-1}-1}{1-\mathsf{t}^{\left(\rho,w^{-1}\eta\right)}X^{-w^{-1}\eta}}\approx \begin{cases} \mathsf{t}^{-1} &\text{if}\ w^{-1}\eta>0,\\ -\mathsf{t}^{-\left(\rho,w^{-1}\eta\right)-1}X^{w^{-1}\eta} & \text{if}\ w^{-1}\eta<0. \end{cases} \end{align} $$

Remark 3.5. These computations are closely related to those of [Reference Brenti, Fomin and Postnikov3, §4] involving quantum Bruhat operators. The precise connection to the quantum Bruhat graph will be made later.

3.3.1. Expanding products as sums over walks

Using the vanishing of the matrix coefficients, we obtain the following row expansion (see Section 2.7) for any $w\in W$ and any sequence $\vec {\eta }=(\eta _1,\dotsc ,\eta _n)\in \left (\Delta ^+\right )^n$ :

(3.42) $$ \begin{align} \varkappa\left(\mathsf{G}_{\eta_1}^\pm\mathsf{G}_{\eta_2}^\pm\dotsm \mathsf{G}_{\eta_n}^\pm\right)_{w,\bullet} &=\sum_{\mathbf{w}\in\mathbf{W}_{w}^{\vec{\eta}}}\varkappa\left(\mathsf{G}_{\eta_1}^\pm\right)_{w_0,w_1}\varkappa\left(\mathsf{G}_{\eta_2}^\pm\right)_{w_1,w_2}\dotsm\varkappa\left(\mathsf{G}_{\eta_n}^\pm\right)_{w_{n-1},w_n}e_{w_n}, \end{align} $$

where $w_0=w$ and $\mathbf {W}_{w}^{\vec {\eta }}$ is the set of sequences $\mathbf {w}=(w_1,\dotsc ,w_n)$ of elements of W such that

(3.43) $$ \begin{align} w_t\in\left\{w_{t-1},s_{\eta_t}w_{t-1}\right\} \text{ for each } t=1,\dotsc,n. \end{align} $$

We call elements of $\mathbf {W}_{w}^{\vec {\eta }}$ walks in W.

3.3.2. Quantum walks

We will see that the matrices describing our inverse Chevalley formula are given by sums over subsets of walks given as follows:

Definition 3.6. We call $\mathbf {w}=(w_1,\dotsc ,w_n)\in \mathbf {W}_{w}^{\vec {\eta }}$ a quantum walk if it defines a directed walk in the quantum Bruhat graph $\mathrm {QBG}(W)$ – that is, for each $1\le t\le n$ , either $w_t=w_{t-1}$ or $w_{t-1}\to w_t=s_{\eta _t}w_{t-1}$ is an edge in $\mathrm {QBG}(W)$ .

We note that such an edge $w_{t-1}\to w_t$ in $\mathrm {QBG}(W)$ is labelled by $\pm w_{t-1}^{-1}\eta _t$ . In the case when $w_{t-1}>w_t$ , the requirement for an edge is that $\ell (w_{t-1})-\ell (w_t)=\left (2\rho ,-w_{t-1}^{-1}\eta _t\right )-1$ .

3.3.3. Main limit

Now we proceed to the limit computation which will give our first main result. Let $\varpi _k\in P_+$ be a minuscule fundamental weight. (Thus we implicitly assume that our simply laced group G is not of type $E_8$ .) As in the introduction, let $J=I\setminus \{k\}$ , let $W_J=\left \langle s_j \mid j\in J\right \rangle \subset W$ be the corresponding parabolic subgroup, which is the stabiliser of $\varpi _k$ , and let $W^J$ denote the set of minimal coset representatives for $W/W_J$ . For $w\in W$ , let $\left \lfloor w \right \rfloor \in W^J$ be the minimal representative of its coset $wW_J$ .

Fix $x\in W^J$ and let $\lambda =x\varpi _k\in P$ , which is an arbitrary minuscule weight. Let $y \in W$ be the unique element such that $\left \lfloor w_{\circ } \right \rfloor = yx$ . Since $\varpi _{k}$ is minuscule, we have $\ell (\left \lfloor w_{\circ } \right \rfloor ) = \ell (y) + \ell (x)$ ; indeed, on $W^J$ , the Bruhat order coincides with the left weak Bruhat order [Reference Green6, Lemma 11.1.16].

We fix reduced expressions $x=s_{j_l}\dotsm s_{j_1}$ and $y = s_{i_{1}} \cdots s_{i_{m}}$ , and we define

(3.44) $$ \begin{align} \beta_r &= s_{j_{l}}s_{j_{l-1}} \cdots s_{j_{r+1}}\left(\alpha_{j_{r}}\right)\kern-2pt, \quad 1 \le r \le l, \end{align} $$

(3.45) $$ \begin{align} \gamma_s &=s_{i_{m}}s_{i_{m-1}} \cdots s_{i_{s+1}}\left(\alpha_{i_{s}}\right)\kern-2.5pt, \quad 1 \le s \le m. \end{align} $$

Hence, if we set

(3.46) $$ \begin{align} \mathrm{Inv}(w)=\Delta^+\cap w\left(-\Delta^+\right)\kern-2pt, \end{align} $$

then $\mathrm {Inv}(x)=\{\beta _1,\dots ,\beta _l\}$ and $\mathrm {Inv}\left (y^{-1}\right )=\{\gamma _{1},\dots ,\gamma _{m}\}$ .

Now, for any $w\in W$ , define the set of walks $\mathbf {W}_{\lambda ,w}=\mathbf {W}_{w}^{\vec {\eta }}$ , where $\vec {\eta }$ is given by

(3.47) $$ \begin{align} \vec{\eta}=(\eta_1,\dotsc,\eta_n)=(\beta_l,\dotsc,\beta_1,\gamma_1,\dotsc,\gamma_m) \end{align} $$

and $n=l+m$ . Let $\mathbf {QW}_{\lambda ,w}$ denote the subset of quantum walks in $\mathbf {W}_{w}^{\vec {\eta }}$ . (These sets depend on the choice of reduced expressions for x and y, even though our notation does not indicate this.)

The inverse Chevalley formula for minuscule weights is an immediate consequence of the following, which is the main technical achievement of this paper:

Theorem 3.7. For any minuscule $\lambda \in P$ and $w\in W$ , we have

(3.48) $$ \begin{align} \varrho_0'\left(Y^{\lambda}\right)_{w,\bullet}=\sum_{\mathbf{w}=(w_1,\dotsc,w_n)\in\mathbf{QW}_{\lambda,w}}g^-_{\mathbf{w}} t_{w_{l}^{-1}\lambda} g^+_{\mathbf{w}} e_{w_n}, \end{align} $$

with

(3.49) $$ \begin{align} g^-_{\mathbf{w}} &= \prod_{1\le t\le l} \begin{cases} 1-X^{-w_{t-1}^{-1}\eta_t} & \text{if}\ w_t=w_{t-1}\ \text{and}\ \left(\rho,w_{t-1}^{-1}\eta_t\right)=1,\\ -X^{w_{t-1}^{-1}\eta_t} & \text{if}\ w_t<w_{t-1},\\ 1 & \text{otherwise} \end{cases} \end{align} $$

(3.50) $$ \begin{align} g^+_{\mathbf{w}} &= \prod_{l<t\le n} \begin{cases} 1-X^{w_{t-1}^{-1}\eta_t} & \text{if}\ w_t=w_{t-1}\ \text{and}\ \left(\rho,w_{t-1}^{-1}\eta_t\right)=-1,\\ X^{w_{t-1}^{-1}\eta_t} & \text{if}\ w_t<w_{t-1},\\ -1 & \text{if}\ w_t>w_{t-1},\\ 1 & \text{otherwise}. \end{cases} \end{align} $$

For the proof of Theorem 3.7, we will need the following notions, which rely heavily on $\lambda \in P$ being minuscule. For $w \in W$ , and with all other notation as before, let us define

(3.51) $$ \begin{align} & \mathrm{Inv}(w)_{\lambda}^{+}=\{ \gamma \in \mathrm{Inv}(w) \mid (\gamma,\lambda)=1\}, \qquad \ell_{\lambda}^{+}(w)=\# \mathrm{Inv}(w)_{\lambda}^{+}, \end{align} $$

(3.52) $$ \begin{align} & \mathrm{Inv}(w)_{\lambda}^{-}=\{ \gamma \in \mathrm{Inv}(w) \mid (\gamma,\lambda)=-1\}, \qquad \ell_{\lambda}^{-}(w)=\# \mathrm{Inv}(w)_{\lambda}^{-}, \end{align} $$

and

(3.53) $$ \begin{align} \mathrm{Inv}(w)_{\lambda,r}^{-}&=\mathrm{Inv}(w) \cap \{\beta_{r},\beta_{r+1},\dots,\beta_{l}\}, \qquad \ell_{\lambda,r}^{-}(w)=\#\mathrm{Inv}(w)_{\lambda,r}^{-}, \end{align} $$

(3.54) $$ \begin{align} \mathrm{Inv}(w)_{\lambda,s}^{+}&=\mathrm{Inv}(w) \cap \{\gamma_{s},\gamma_{s+1},\dots,\gamma_{m}\}, \qquad \ell_{\lambda,s}^{+}(w)=\#\mathrm{Inv}(w)_{\lambda,s}^{+}, \end{align} $$

where $1 \le r \le l$ and $1 \le s \le m$ .

The following lemma seems to be well known, but we give a simple proof in Appendix B for the convenience of the reader:

Lemma 3.8. With all notation as before, the following are true:

  1. 1. We have

    (3.55) $$ \begin{align} 2\ell(x) - \ell(\left\lfloor w_{\circ} \right\rfloor) &= - 2(\rho,\lambda). \end{align} $$
  2. 2. For any $w\in W$ ,

    (3.56) $$ \begin{align} (\rho-w\rho,\lambda) &= \ell_{\lambda}^{+}(w) - \ell_{\lambda}^{-}(w) \end{align} $$

    and

    (3.57) $$ \begin{align} \mathrm{Inv}(w)_{\lambda}^{-} &= \mathrm{Inv}(w) \cap \mathrm{Inv}(x), \end{align} $$

    (3.58) $$ \begin{align} \mathrm{Inv}(w)_{\lambda}^{+} &= \mathrm{Inv}(w) \cap \mathrm{Inv}\left(y^{-1}\right). \end{align} $$

The proof of the following proposition will be given in Appendix B:

Proposition 3.9. With all notation as before, the following are true:

  1. 1. If $s_{\beta _{r}}w < w$ for $1 \le r \le l$ , then

    (3.59) $$ \begin{align} \ell(w)-\ell\left(s_{\beta_{r}}w\right) = 2 \left(\ell_{\lambda,r}^{-}(w)-\ell_{\lambda,r}^{-}\left(s_{\beta_{r}}w\right)\right) - 1. \end{align} $$
  2. 2. If $s_{\gamma _{s}}w < w$ for $1 \le s \le m$ , then

    (3.60) $$ \begin{align} \ell(w)-\ell\left(s_{\gamma_{s}}w\right) = 2 \left(\ell_{\lambda,s}^{+}(w)-\ell_{\lambda,s}^{+}\left(s_{\gamma_{s}}w\right)\right)- 1. \end{align} $$

Remark 3.10. Equations (3.59) and (3.60) generalise the following well-known formula for length differences in the symmetric group $S_n$ :

$$ \begin{align*} \ell(w)-\ell((ij)w) &= 1+2\cdot\left\lvert\left\{k: i<k<j \text{ and } w^{-1}(i)>w^{-1}(k)>w^{-1}(j)\right\}\right\rvert, \end{align*} $$

where $1\le i < j\le n$ , $w\in S_n$ is a permutation such that $w^{-1}(i)>w^{-1}(j)$ , and $(ij)$ denotes the transposition swapping i and j.

Proof of Theorem 3.7. Let $\pi \in \Pi $ be the length $0$ element of $W_{\mathrm {ex}}$ corresponding to coset $\varpi _k+Q$ in $P/Q$ . We have the formula $t_{\varpi _k} = \pi \left \lfloor w_{\circ } \right \rfloor =\pi yx$ (which may be taken as the definition of $\pi $ ), and hence $t_{\lambda } = x\pi y.$

Using equations (3.33) and (3.55), we find

$$ \begin{align*} Y^{\lambda} &= \mathsf{v}^{2\ell(x)-\ell\left(\left\lfloor w_{\circ} \right\rfloor\right)} T_{x^{-1}}^{-1} \pi T_y\\ &= \mathsf{t}^{-\left(\rho,\lambda\right)}T_{x^{-1}}^{-1} \pi T_y \end{align*} $$

in $\widehat {\mathbb {H}}$ , and

$$ \begin{align*} \varrho\left(Y^{\lambda}\right) &= \mathsf{t}^{-\left(\rho,\lambda\right)} \varrho\left(T_{x^{-1}}^{-1}\right)x^{-1} t_{\lambda}y^{-1} \varrho\left(T_y\right)\\ &= \mathsf{t}^{-\left(\rho,\lambda\right)}\mathsf{G}^-_{\beta_l}\dotsm \mathsf{G}^-_{\beta_1}t_{\lambda}\mathsf{G}^+_{\gamma_1}\dotsm \mathsf{G}^+_{\gamma_m} \end{align*} $$

in the polynomial representation. Thus we need to compute the limit

$$ \begin{align*} \varrho_0'\left(Y^{\lambda}\right)=\lim_{\mathsf{v}\to 0} \varkappa\left(\mathsf{G}^-_{\beta_l}\dotsm \mathsf{G}^-_{\beta_1}\right)\mathsf{t}^{-\left(\rho,\lambda\right)}\varkappa(t_{\lambda})\varkappa\left(\mathsf{G}^+_{\gamma_1}\dotsm \mathsf{G}^+_{\gamma_m}\right). \end{align*} $$

The matrix $\mathsf {t}^{-\left (\rho ,\lambda \right )}\varkappa (t_{\lambda })$ is diagonal with entries

$$ \begin{align*} \mathsf{t}^{-\left(\rho,\lambda\right)}\varkappa(t_{\lambda})_{w,w} = \mathsf{t}^{\left(\rho,w^{-1}\lambda\right)-\left(\rho,\lambda\right)}t_{w^{-1}\lambda}. \end{align*} $$

Using equation (3.56), we can write $\mathsf {t}^{-\left (\rho ,\lambda \right )}\varkappa (t_{\lambda })=\varkappa _\lambda ^-\tau _\lambda \varkappa _\lambda ^+$ , where $\varkappa _\lambda ^\pm $ and $\tau _\lambda $ are the diagonal matrices given by

(3.61) $$ \begin{align} \left(\varkappa_\lambda^-\right)_{w,w}&=\mathsf{t}^{\ell_\lambda^-(w)},\qquad (\tau_\lambda)_{w,w} = t_{w^{-1}\lambda},\qquad \left(\varkappa_\lambda^+\right)_{w,w}=\mathsf{t}^{-\ell_\lambda^+(w)}. \end{align} $$

Our strategy is to commute $\varkappa _\lambda ^-$ to the left past $\varkappa \left (\mathsf {G}^-_{\beta _l}\dotsm \mathsf {G}^-_{\beta _1}\right )$ , and $\varkappa _\lambda ^+$ to the right past $\varkappa \left (\mathsf {G}^+_{\gamma _1}\dotsm \mathsf {G}^+_{\gamma _m}\right )$ . After we have done so, all negative powers of $\mathsf {v}$ will disappear.

For $r=1,\dotsc ,l$ , we have

$$ \begin{align*} \varkappa\left(\mathsf{G}_{\beta_r}^-\right)_{w,w} \cdot \mathsf{t}^{\ell_{\lambda,r}^-(w)} &\approx \mathsf{t}^{\ell_{\lambda,r+1}^-(w)}\times \begin{cases} 1-X^{-w^{-1}\beta_r} & \text{if}\ \left(\rho,w^{-1}\beta_r\right)=1,\\ 1 & \text{if}\ \left(\rho,w^{-1}\beta_r\right)>1,\\ 1 & \text{if}\ w^{-1}\beta_r<0, \end{cases} \end{align*} $$

and

$$ \begin{align*} &\varkappa\left(\mathsf{G}_{\beta_r}^-\right)_{w,s_{\beta_r}w} \cdot \mathsf{t}^{\ell_{\lambda,r}^-\left(s_{\beta_r}w\right)}\\ &\approx \mathsf{t}^{\ell_{\lambda,r+1}^-(w)} \times \begin{cases} \mathsf{t}^{-1+\ell_{\lambda,r}^-\left(s_{\beta_r}w\right)-\ell_{\lambda,r}^-(w)} &\text{if}\ w^{-1}\beta_r>0,\\ -\mathsf{t}^{-\left(\rho,w^{-1}\beta_r\right)+\ell_{\lambda,r}^-\left(s_{\beta_r}w\right)-\ell_{\lambda,r}^-(w)}X^{w^{-1}\beta_r} & \text{if}\ w^{-1}\beta_r<0 \end{cases}\\ &= \mathsf{t}^{\ell_{\lambda,r+1}^-(w)} \times \begin{cases} \mathsf{v}^{-1+\ell\left(s_{\beta_r}w\right)-\ell(w)} &\text{if}\ w^{-1}\beta_r>0,\\ -\mathsf{v}^{-\left(2\rho,w^{-1}\beta_r\right)-1+\ell\left(s_{\beta_r}w\right)-\ell(w)}X^{w^{-1}\beta_r} & \text{if}\ w^{-1}\beta_r<0, \end{cases} \end{align*} $$

where we use equations (3.59) to obtain the last equality. Notice that in all cases, each factor after the bracket involves no negative powers of $\mathsf {v}$ . Moreover, after commuting $\varkappa _\lambda ^-$ all the way to the left, we arrive at $\mathsf {t}^{\ell _{\lambda ,l+1}^-(w)}=1$ , for any $w\in W$ .

Similarly, for $s=1,\dotsc ,m$ , we have

$$ \begin{align*} \mathsf{t}^{-\ell_{\lambda,s}^+(w)}\cdot \varkappa\left(\mathsf{G}_{\gamma_s}^+\right)_{w,w} & \approx \mathsf{t}^{-\ell_{\lambda,s+1}^+(w)}\times \begin{cases} 1 & \text{if}\ w^{-1}\gamma_s>0,\\ 1-X^{w^{-1}\gamma_s} & \text{if}\ \left(\rho,w^{-1}\gamma_s\right)=-1,\\ 1 & \text{if}\ \left(\rho,w^{-1}\gamma_s\right)<-1, \end{cases} \end{align*} $$

and

$$ \begin{align*} &\mathsf{t}^{-\ell_{\lambda,s}^+(w)}\cdot \varkappa\left(\mathsf{G}_{\gamma_s}^+\right)_{w,s_{\gamma_s} w}\\ &\approx \mathsf{t}^{-\ell_{\lambda,s+1}^+(s_{\gamma_s}w)} \times \begin{cases} -\mathsf{t}^{-1+\ell_{\lambda,s}^+\left(s_{\gamma_s}w\right)-\ell_{\lambda,s}^+(w)}&\text{if}\ w^{-1}\gamma_s>0,\\ \mathsf{t}^{-\left(\rho,w^{-1}\gamma_s\right)+\ell_{\lambda,s}^+\left(s_{\gamma_s}w\right)-\ell_{\lambda,s}^+(w)}X^{w^{-1}\gamma_s} & \text{if}\ w^{-1}\gamma_s<0 \end{cases}\\ &= \mathsf{t}^{-\ell_{\lambda,s+1}^+\left(s_{\gamma_s}w\right)} \times \begin{cases} -\mathsf{v}^{-1+\ell\left(s_{\gamma_s}w\right)-\ell(w)} & \text{if}\ w^{-1}\gamma_s>0,\\ \mathsf{v}^{-\left(2\rho,w^{-1}\gamma_s\right)-1+\ell\left(s_{\gamma_s}w\right)-\ell(w)}X^{w^{-1}\gamma_s} & \text{if}\ w^{-1}\gamma_s<0. \end{cases} \end{align*} $$

As before, each factor after the bracket involves no negative powers of $\mathsf {v}$ , and after commuting $\varkappa _\lambda ^+$ all the way to the right, we arrive at $\mathsf {t}^{\ell _{\lambda ,m+1}^+(w)}=1$ , for any $w\in W$ .

To complete the proof, we expand the product of resulting matrices (including $\tau _\lambda $ ) and take $\mathsf {v}\to 0$ . Taking into account the exponents of $\mathsf {v}$ , one sees that the surviving terms are exactly those indexed by quantum walks $\mathbf {QW}_{\lambda ,w}$ .□

3.4. Inverse Chevalley formula

By means of [Reference Orr25, Theorem 5.1], Theorem 3.7 immediately gives the following algebraic form of the inverse Chevalley formula in $K_{H\times \mathbb {C}^*}(\mathbf {Q}_{G})$ , which is our first main result:

Theorem 3.11. For any minuscule $\lambda \in P$ and $w\in W$ , we have

(3.62) $$ \begin{align} e^\lambda\cdot\left[\mathcal{O}_{\mathbf{Q}_{G}(w)}\right] = \sum_{\mathbf{w}=(w_1,\dotsc,w_n)\in\mathbf{QW}_{\lambda,w}}\left[\mathcal{O}_{\mathbf{Q}_{G}(w_n)}\right]\cdot \widetilde{g}^+_{\mathbf{w}} X^{-w_{\circ} w_l^{-1}\lambda} \widetilde{g}^-_{\mathbf{w}} \end{align} $$

in terms of the q-Heisenberg action of Proposition 3.3, with

(3.63) $$ \begin{align} \widetilde{g}^-_{\mathbf{w}} &= \prod_{1\le t\le l} \begin{cases} 1-\widetilde{X}^{-w_{\circ} w_{t-1}^{-1}\eta_t} & \text{if}\ w_t=w_{t-1}\ \text{and}\ \left(\rho,w_{t-1}^{-1}\eta_t\right)=1,\\ -\widetilde{X}^{w_{\circ} w_{t-1}^{-1}\eta_t} & \text{if}\ w_t<w_{t-1},\\ 1 & \text{otherwise} \end{cases} \end{align} $$

(3.64) $$ \begin{align} \widetilde{g}^+_{\mathbf{w}} &= \prod_{l< t\le n} \begin{cases} 1-\widetilde{X}^{w_{\circ} w_{t-1}^{-1}\eta_t} & \text{if}\ w_t=w_{t-1}\ \text{and}\ \left(\rho,w_{t-1}^{-1}\eta_t\right)=-1,\\ \widetilde{X}^{w_{\circ} w_{t-1}^{-1}\eta_t} & \text{if}\ w_t<w_{t-1},\\ -1 & \text{if}\ w_t>w_{t-1},\\ 1 & \text{otherwise}, \end{cases} \end{align} $$

and $\widetilde {X}^\beta =q\cdot t_\beta X^\beta $ for $\beta \in \Delta $ .

Remark 3.12. The elements $\widetilde {X}^\beta $ commute, as one easily checks.

Remark 3.13. The image of equation (3.62) under the map $\mathrm {trunc}$ of formula (1.3) recovers the corresponding inverse Chevalley formula in $K_H(G/B)$ given by [Reference Mathieu23, p. 239]; note that under the map $\mathrm {trunc}$ , the class $\left [\mathcal {O}_{\mathbf {Q}_{G}}(\mu )\right ]$ for $\mu \in P$ corresponds to the class $\left [\mathcal {O}_{G/B}(\mu )\right ]$ , which is denoted by $[\mathcal {L}(-\mu )]$ in [Reference Mathieu23]. We note also that the image of equation (3.62) under $\mathrm {trunc}$ has positive coefficients for $\lambda \in -P_+$ (i.e., when $l=n$ ), and alternating coefficients for $\lambda \in P_+$ (i.e., when $l=0$ ). Informally, the effect of the map $\mathrm {trunc}$ is achieved by setting $\widetilde {X}^\beta =0$ in equations (3.63) and (3.64), where $\beta \in \Delta ^+$ always.

3.4.1. Decorated quantum walks

We now proceed to further combinatorialise equation (3.62). First we enlarge the summation set, so that each term on the right-hand side of equation (3.62) is a product of monomials in the q-Heisenberg algebra.

Given $\mathbf {w}=(w_1,\dotsc ,w_n)\in \mathbf {QW}_{\lambda ,w}$ , let $S^-(\mathbf {w})$ denote the set of steps t, for $1\le t\le l$ , such that $w_t=w_{t-1}$ and $\left (\rho ,w_{t-1}^{-1}\eta _t\right )=1$ . Similarly, let $S^+(\mathbf {w})$ denote the set of steps t, for $l<t\le n$ such that $w_t=w_{t-1}$ and $\left (\rho ,w_{t-1}^{-1}\eta _t\right )=-1$ .

Let $S(\mathbf {w})=S^-(\mathbf {w})\cup S^+(\mathbf {w})$ , and define the set of decorated quantum walks $\widetilde {\mathbf {QW}}_{\lambda ,w}$ to consist of all pairs $(\mathbf {w},\mathbf {b})$ where $\mathbf {w}\in \mathbf {QW}_{\lambda ,w}$ and $\mathbf {b}$ is a $\{0,1\}$ -valued function on $S(\mathbf {w})$ . Then equation (3.62) can be written as

(3.65) $$ \begin{align} e^{\lambda}\cdot\left[\mathcal{O}_{\mathbf{Q}_{G}(w)}\right] = \sum_{\left(\mathbf{w},\mathbf{b}\right)\in\widetilde{\mathbf{QW}}_{\lambda,w}}\left[\mathcal{O}_{\mathbf{Q}_{G}(w_n)}\right]\cdot \widetilde{g}^+_{\left(\mathbf{w},\mathbf{b}\right)} X^{-w_{\circ} w_l^{-1}\lambda} \widetilde{g}^-_{\left(\mathbf{w},\mathbf{b}\right)}, \end{align} $$

with

(3.66) $$ \begin{align} \widetilde{g}^-_{\left(\mathbf{w},\mathbf{b}\right)} &= \prod_{1\le t\le l} \begin{cases} \left(-\widetilde{X}^{-w_{\circ} w_{t-1}^{-1}\eta_t}\right)^{\mathbf{b}(t)} & \text{if}\ t\in S^-(\mathbf{w}),\\ -\widetilde{X}^{w_{\circ} w_{t-1}^{-1}\eta_t} & \text{if}\ w_t<w_{t-1},\\ 1 & \text{otherwise} \end{cases} \end{align} $$

(3.67) $$ \begin{align} \widetilde{g}^+_{\left(\mathbf{w},\mathbf{b}\right)} &= \prod_{l< t\le n} \begin{cases} \left(-\widetilde{X}^{w_{\circ} w_{t-1}^{-1}\eta_t}\right)^{\mathbf{b}(t)} & \text{if}\ t\in S^+(\mathbf{w}),\\ \widetilde{X}^{w_{\circ} w_{t-1}^{-1}\eta_t} & \text{if}\ w_t<w_{t-1},\\ -1 & \text{if}\ w_t>w_{t-1},\\ 1 & \text{otherwise}. \end{cases} \end{align} $$

Next, to expand equation (3.65) further, let us introduce some more notation. For $(\mathbf {w},\mathbf {b})\in \widetilde {\mathbf {QW}}_{\lambda ,w}$ , define the sign

$$ \begin{align*} (-1)^{\left(\mathbf{w},\mathbf{b}\right)} = \prod_{\substack{1\le t\le l\\w_t<w_{t-1}}}(-1)\prod_{\substack{l< t\le n \\ w_t>w_{t-1}}}(-1)\prod_{t\in S(\mathbf{w})}(-1)^{\mathbf{b}(t)} \end{align*} $$

and partial weights (in Q)

$$ \begin{align*} \mathrm{wt}_0(\mathbf{w},\mathbf{b})&=0\\ \mathrm{wt}_t(\mathbf{w},\mathbf{b})&=\mathrm{wt}_{t-1}(\mathbf{w},\mathbf{b})+ \begin{cases} -\mathbf{b}(t)w_{\circ} w_{t-1}^{-1}\eta_t & \text{if}\ t\in S^-(\mathbf{w}),\\ w_{\circ} w_{t-1}^{-1}\eta_t & \text{if}\ w_t<w_{t-1},\\ 0 & \text{otherwise} \end{cases} \quad \text{for}\ 1\le t\le l\\ \mathrm{wt}_t(\mathbf{w},\mathbf{b})&=\mathrm{wt}_{t-1}(\mathbf{w},\mathbf{b})+ \begin{cases} \mathbf{b}(t)w_{\circ} w_{t-1}^{-1}\eta_t & \text{if}\ t\in S^+(\mathbf{w}),\\ w_{\circ} w_{t-1}^{-1}\eta_t & \text{if}\ w_t<w_{t-1},\\ 0 & \text{otherwise} \end{cases} \quad \text{for}\ l<t\le n. \end{align*} $$

Define the weight of $(\mathbf {w},\mathbf {b})$ to be $\mathrm {wt}(\mathbf {w},\mathbf {b})=\mathrm {wt}_n(\mathbf {w},\mathbf {b})\in Q$ , and set $d_t(\mathbf {w},\mathbf {b})=\mathrm {wt}_t(\mathbf {w},\mathbf {b})-\mathrm {wt}_{t-1}(\mathbf {w},\mathbf {b})$ for $1\le t\le n$ .

Define the partial degrees (in $\mathbb {Z}$ ):

$$ \begin{align*} \deg_0^-(\mathbf{w},\mathbf{b}) &= 0\\ \deg_t^-(\mathbf{w},\mathbf{b}) &= \deg_{t-1}^-(\mathbf{w},\mathbf{b})+\frac{\left\lvert d_t(\mathbf{w},\mathbf{b})\right\rvert^2}{2}+(d_t(\mathbf{w},\mathbf{b}),\mathrm{wt}_{t-1}(\mathbf{w},\mathbf{b})),\quad \text{for}\ 1\le t\le l\\ \deg_{l}^+(\mathbf{w},\mathbf{b}) &= \deg_{l}^-(\mathbf{w},\mathbf{b})+\left(-w_{\circ} w_{l}^{-1}\lambda,\mathrm{wt}_{l}(\mathbf{w},\mathbf{b})\right)\\ \deg_t^+(\mathbf{w},\mathbf{b}) &= \deg_{t-1}^+(\mathbf{w},\mathbf{b})+\frac{\left\lvert d_t(\mathbf{w},\mathbf{b})\right\rvert^2}{2}+(d_t(\mathbf{w},\mathbf{b}),\mathrm{wt}_{t-1}(\mathbf{w},\mathbf{b})), \quad \text{for}\ l<t\le n. \end{align*} $$

Define $\deg (\mathbf {w},\mathbf {b})=\deg _n^+(\mathbf {w},\mathbf {b})\in \mathbb {Z}$ .

Then, from equation (3.65), we obtain our second main result, the combinatorial form of our inverse Chevalley formula:

Theorem 3.14. For any minuscule $\lambda \in P$ and $w\in W$ , we have

(3.68) $$ \begin{align} &e^\lambda\cdot\left[\mathcal{O}_{\mathbf{Q}_{G}(w)}\right]\nonumber\\ &\quad= \sum_{\left(\mathbf{w},\mathbf{b}\right)\in\widetilde{\mathbf{QW}}_{\lambda,w}}(-1)^{\left(\mathbf{w},\mathbf{b}\right)}q^{\deg\left(\mathbf{w},\mathbf{b}\right)}\cdot\left[\mathcal{O}_{\mathbf{Q}_{G}\left(w_nt_{-w_{\circ}\left(\mathrm{wt}\left(\mathbf{w},\mathbf{b}\right)\right)}\right)}\left(-w_{\circ} w_l^{-1}\lambda+\mathrm{wt}(\mathbf{w},\mathbf{b})\right)\right]. \end{align} $$

Proof. Proceed from right to left in equation (3.65), commuting all translations to the left of line bundle twists, and then act on $\left [\mathcal {O}_{\mathbf {Q}_{G}(w_n)}\right ]$ .□

Appendix A The type A case

We briefly consider the type A case. For this appendix, let $G=SL(n+1,\mathbb {C})$ .

A.1. The root system of type A

Let $\{\varepsilon _i : 1\le i\le n+1\}$ be the standard basis of $\mathbb {Z}^{n+1}$ . We realise the weight lattice as $P=\mathbb {Z}^{n+1}/\mathbb {Z}(\varepsilon _1+\dotsm +\varepsilon _{n+1})$ , and by abuse of notation, we continue to denote the image of $\varepsilon _i$ in P by the same symbol. Thus $\varpi _k := \varepsilon _1+\dotsm +\varepsilon _k$ , for $k \in \{1, \ldots , n\}$ , are the fundamental weights of G.

We set $\alpha _{i} := \varepsilon _{i} - \varepsilon _{i+1}$ for $i \in \{ 1, \ldots , n \}$ and $\alpha _{i, j} := \alpha _{i} + \alpha _{i+1} + \cdots + \alpha _{j}$ for $i, j \in \{1, \ldots , n\}$ with $i \le j$ . Then the root system is $\Delta := \left \{ \pm \alpha _{i, j} \mid 1 \le i \le j \le n \right \}$ , with the set of positive roots $\Delta ^+ := \left \{ \alpha _{i, j} \mid 1 \le i \le j \le n \right \}$ and the set of simple roots $\{ \alpha _{1}, \ldots , \alpha _{n} \}$ .

We identify the Weyl group W with the symmetric group $\mathfrak {S}_{n+1}$ in the usual way. Regarding $w\in W$ as a permutation, we have $w \varepsilon _{i} = \varepsilon _{w(i)}$ for $i \in \{ 1, \ldots , n+1 \}$ . Thus, for $i, j \in \{ 1, \ldots , n \}$ with $i \leq j$ , the reflection $s_{\alpha _{i,j}}$ corresponds to the transposition $(i, j+1)\in \mathfrak {S}_{n+1}$ . The longest element of W is given by $w_{\circ }(i) = n+2-i$ for $i \in \{ 1, \ldots , n+1 \}$ .

A.2. Generators

Each fundamental weight $\varpi _k$ ( $k=1,\dotsc ,n$ ) is minuscule. Moreover, we have $w_{\circ } \varpi _k=-\varpi _{n+1-k}$ . Since the $\varepsilon _i$ generate P, the inverse Chevalley rule in $K_{H\times \mathbb {C}^*}\big(\mathbf {Q}_{G}^{\mathrm {rat}}\big )$ is completely determined by Theorem 3.14 in the case of $\lambda =\pm \varepsilon _i$ , where $1\le i\le n+1$ . (That is, $\lambda $ belongs to the W-orbit of either $\varpi _1$ or $\varpi _n=-\varepsilon _{n+1}$ .)

A.3. Examples

We give two concrete examples of our inverse Chevalley formula (3.62) in type A.

First, for $G=SL(3,\mathbb {C})$ and $\lambda =\varpi _1$ , we have

(A.1) $$ \begin{align} &e^{\varpi_1}\cdot\left[\mathcal{O}_{\mathbf{Q}_{G}\left(s_1\right)}\right] = \left[\mathcal{O}_{\mathbf{Q}_{G}\left(s_1\right)}(-w_{\circ} s_1\varpi_1)\right] -q\left[\mathcal{O}_{\mathbf{Q}_{G}\left(s_1t_{\alpha_1}\right)}(-w_{\circ}\varpi_1)\right]\nonumber\\ &\qquad\qquad\qquad\qquad-\left[\mathcal{O}_{\mathbf{Q}_{G}\left(s_1s_2\right)}(-w_{\circ} s_1\varpi_1)\right] +q\left[\mathcal{O}_{\mathbf{Q}_{G}\left(t_{\alpha_1}\right)}(-w_{\circ}\varpi_1)\right]. \end{align} $$

Indeed, $\left \lfloor w_{\circ } \right \rfloor =s_2s_1$ , $l=0$ , $\vec {\eta }=(\gamma _1,\gamma _2)=(\alpha _1+\alpha _2,\alpha _1)$ and three of the four possible walks are quantum walks, namely

$$ \begin{align*}\mathbf{QW}_{\varpi_1,s_1}=\{(s_1,s_1),(s_1 s_2,s_1 s_2),(s_1,e)\}.\end{align*} $$

(The reason the remaining walk $(s_1s_2,s_2)\in \mathbf {W}_{\varpi _1,s_1}$ is not a quantum walk is that $s_1s_2\to s_2$ is not an edge in $\mathrm {QBG}(W)$ .) The right-hand side of equation (3.62) produces the following expression for $e^{\varpi _1}\cdot \left [\mathcal {O}_{\mathbf {Q}_{G}\left (s_1\right )}\right ]$ :

$$ \begin{align*} \left(\left[\mathcal{O}_{\mathbf{Q}_{G}\left(s_1\right)}\right]\cdot \widetilde{g}^+_{\left(s_1,s_1\right)}+\left[\mathcal{O}_{\mathbf{Q}_{G}\left(s_1s_2\right)}\right]\cdot \widetilde{g}^+_{\left(s_1s_2,s_1s_2\right)}+\left[\mathcal{O}_{\mathbf{Q}_{G}(e)}\right]\cdot \widetilde{g}^+_{\left(s_1,e\right)}\right)\cdot X^{-w_\circ s_1\varpi_1}, \end{align*} $$

where the q-Heisenberg elements are

$$ \begin{align*} \widetilde{g}^+_{\left(s_1,s_1\right)} &= 1-\widetilde{X}^{w_\circ s_1\alpha_1}=1-qt_{-w_\circ\alpha_1}X^{-w_\circ\alpha_1},\\ \widetilde{g}^+_{\left(s_1s_2,s_1s_2\right)} &= -1,\\ \widetilde{g}^+_{\left(s_1,e\right)} &=\widetilde{X}^{w_\circ s_1\alpha_1}=qt_{-w_\circ\alpha_1}X^{-w_\circ\alpha_1}. \end{align*} $$

Finally, we use the formulas in Proposition 3.3 to obtain equation (A.1).

For our second example, we consider $G=SL(4,\mathbb {C})$ and $\lambda =s_2\varpi _2=\varepsilon _1+\varepsilon _3$ . One has $\left \lfloor w_{\circ } \right \rfloor =s_2s_1s_3s_2$ , $l=1$ and $\vec {\eta }=(\beta _1,\gamma _1,\gamma _2,\gamma _3)=(\alpha _2,\alpha _1+\alpha _2+\alpha _3, \alpha _1,\alpha _3)$ . Using one-line notation for permutations in $W=S_4$ , equation (3.62) gives

$$ \begin{align*} e^{\varepsilon_1+\varepsilon_3}\cdot\left[\mathcal{O}_{\mathbf{Q}_{G}(4312)}\right] ={} & \left[\mathcal{O}_{\mathbf{Q}_{G}(4312)}\right]\cdot \left(1-\widetilde{X}^{-w_{\circ}\alpha_1}\right)X^{-w_{\circ} w^{-1}\lambda}\\ & + \left[\mathcal{O}_{\mathbf{Q}_{G}(3412)}\right]\cdot \widetilde{X}^{-w_{\circ}\alpha_1} X^{-w_{\circ} w^{-1}\lambda}\\ & - \left[\mathcal{O}_{\mathbf{Q}_{G}(4321)}\right]\cdot \left(1-\widetilde{X}^{-w_{\circ}\alpha_1}\right)X^{-w_{\circ} w^{-1}\lambda}\\ & - \left[\mathcal{O}_{\mathbf{Q}_{G}(3421)}\right]\cdot \widetilde{X}^{-w_{\circ}\alpha_1} X^{-w_{\circ} w^{-1}\lambda}\\ & + \left[\mathcal{O}_{\mathbf{Q}_{G}(1342)}\right]\cdot \widetilde{X}^{-w_{\circ}\left(\alpha_1+\alpha_2\right)} X^{-w_{\circ} w^{-1}\lambda}\\ & - \left[\mathcal{O}_{\mathbf{Q}_{G}(1432)}\right]\cdot \widetilde{X}^{-w_{\circ}\left(\alpha_1+\alpha_2\right)} X^{-w_{\circ} w^{-1}\lambda}\\ & - \left[\mathcal{O}_{\mathbf{Q}_{G}(2341)}\right]\cdot \widetilde{X}^{-w_{\circ}\left(\alpha_1+\alpha_2\right)} X^{-w_{\circ} w^{-1}\lambda}\\ & + \left[\mathcal{O}_{\mathbf{Q}_{G}(2431)}\right]\cdot \widetilde{X}^{-w_{\circ}\left(\alpha_1+\alpha_2\right)} X^{-w_{\circ} w^{-1}\lambda}, \end{align*} $$

where $w=4312$ . We note that $\widetilde {g}^-_{\mathbf {w}}=1$ and $w_l=w$ for all quantum walks in $\mathbf {QW}_{\varepsilon _1+\varepsilon _2,4312}$ . Using Proposition 3.3, we obtain

(A.2) $$ \begin{align} e^{\varepsilon_1+\varepsilon_3}\cdot\left[\mathcal{O}_{\mathbf{Q}_{G}(4312)}\right] ={} & \left[\mathcal{O}_{\mathbf{Q}_{G}(4312)}\left(-w_{\circ} w^{-1}\lambda\right)\right]-q\left[\mathcal{O}_{\mathbf{Q}_{G}\left(4312\cdot t_{\alpha_1}\right)}\left(-w_{\circ}\left(w^{-1}\lambda+\alpha_1\right)\right)\right]\notag\\ & + q\left[\mathcal{O}_{\mathbf{Q}_{G}\left(3412\cdot t_{\alpha_1}\right)}\left(-w_{\circ}\left(w^{-1}\lambda+\alpha_1\right)\right)\right]\notag\\ & - \left[\mathcal{O}_{\mathbf{Q}_{G}(4321)}\left(-w_{\circ} w^{-1}\lambda\right)\right]+q\left[\mathcal{O}_{\mathbf{Q}_{G}\left(4321\cdot t_{\alpha_1}\right)}\left(-w_{\circ}\left(w^{-1}\lambda+\alpha_1\right)\right)\right] \notag\\ & - q\left[\mathcal{O}_{\mathbf{Q}_{G}\left(3421\cdot t_{\alpha_1}\right)}\left(-w_{\circ}\left(w^{-1}\lambda+\alpha_1\right)\right)\right]\notag\\ & + q\left[\mathcal{O}_{\mathbf{Q}_{G}\left(1342\cdot t_{\alpha_1+\alpha_2}\right)}\left(-w_{\circ}\left(w^{-1}\lambda+\alpha_1+\alpha_2\right)\right)\right]\notag\\ & - q\left[\mathcal{O}_{\mathbf{Q}_{G}\left(1432\cdot t_{\alpha_1+\alpha_2}\right)}\left(-w_{\circ}\left(w^{-1}\lambda+\alpha_1+\alpha_2\right)\right)\right]\notag\\ & - q\left[\mathcal{O}_{\mathbf{Q}_{G}\left(2341\cdot t_{\alpha_1+\alpha_2}\right)}\left(-w_{\circ}\left(w^{-1}\lambda+\alpha_1+\alpha_2\right)\right)\right]\notag\\ & + q\left[\mathcal{O}_{\mathbf{Q}_{G}\left(2431\cdot t_{\alpha_1+\alpha_2}\right)}\left(-w_{\circ}\left(w^{-1}\lambda+\alpha_1+\alpha_2\right)\right)\right], \end{align} $$

where $w^{-1}\lambda = \varepsilon _2+\varepsilon _3$ , $w^{-1}\lambda +\alpha _1 = \varepsilon _1+\varepsilon _3$ and $w^{-1}\lambda +\alpha _1+\alpha _2 = \varepsilon _1+\varepsilon _2$ .

A.4. Special case: $w=w_{\circ }$ and $k=1$

For $G=SL(n+1,\mathbb {C})$ , we consider the special case $w=w_{\circ }$ and $k=1$ ; the latter means that $\lambda =\varepsilon _{l+1}$ for some $0\leq l\leq n$ . In the setting of Section 3.3.3, we have

(A.3) $$ \begin{align} \left\lfloor w_{\circ} \right\rfloor&=s_n \dotsm s_1 \end{align} $$

(A.4) $$ \begin{align} &\qquad\qquad\qquad =\underbrace{s_{n}\dotsm s_{l+1}}_{=y}\underbrace{s_l\dotsm s_1}_{=x} \end{align} $$

and $\vec {\eta }=(\eta _1,\dotsc ,\eta _n)=(\beta _l,\dotsc ,\beta _1,\gamma _1,\dotsc ,\gamma _m)$ , where $m=\ell (y)=n-l$ and

(A.5) $$ \begin{align} \beta_r &=\varepsilon_r-\varepsilon_{l+1},\quad 1\le r\le l, \end{align} $$

(A.6) $$ \begin{align} \qquad \gamma_s &=\varepsilon_{l+1} - \varepsilon_{n+2-s},\quad 1\le s\le m. \end{align} $$

Let us abbreviate our notation as follows: $\mathbf {W}_w^{(l)}=\mathbf {W}_{\varepsilon _{l+1},w}$ and $\mathbf {QW}_w^{(l)}=\mathbf {QW}_{\varepsilon _{l+1},w}$ .

The following lemma describes the set $\mathbf {QW}^{(l)}_{w_\circ }$ . Its proof is straightforward and is left to the reader.

Lemma A.1. A walk $\mathbf {w}\in \mathbf {W}_{w_\circ }^{(l)}$ belongs to $\mathbf {QW}^{(l)}_{w_\circ }$ if and only if one of the following holds:

  1. (a) $\mathbf {w}=(w_1,\dotsc ,w_l,w_{l},\dotsc ,w_{l})$ , where $(w_1,\dotsc ,w_{l})\in \mathbf {W}_{w_\circ }^{\left (\alpha _{l,l},\dotsc ,\alpha _{1,l}\right )}$ .

  2. (b) $\mathbf {w}=(w_\circ ,\dotsc ,w_\circ ,w_{l+1},\dotsc ,w_n)$ , where $(w_{l+1},\dotsc ,w_n)\in \mathbf {W}_{w_\circ }^{\left (\alpha _{l+1,n},\dotsc ,\alpha _{l+1,l+1}\right )}$ .

We have $S^-(\mathbf {w})=\varnothing $ in all cases, and $S^+(\mathbf {w})=\varnothing $ unless $l<n$ and $\mathbf {w}=(w_\circ ,\dotsc ,w_\circ )$ , in which case $S^+(\mathbf {w})=\{n\}$ . (Note that the two cases in the lemma share the walk $\mathbf {w}=(w_\circ ,\dotsc ,w_\circ )$ .)

Combining Lemma A.1 with Theorem 3.11, one obtains the following (where we set $i=l+1$ ):

Proposition A.2. In $K_{H\times \mathbb {C}^*}(\mathbf {Q}_{G})$ for $G=SL(n+1,\mathbb {C})$ , one has

(A.7) $$ \begin{align} e^{\varepsilon_{i}}\cdot\left[\mathcal{O}_{\mathbf{Q}_{G}\left(w_\circ\right)}\right] ={}& \left[\mathcal{O}_{\mathbf{Q}_{G}\left(w_\circ\right)}(-\varepsilon_{i})\right]-\mathbf{1}_{\{i<n+1\}}\cdot q\cdot \left[\mathcal{O}_{\mathbf{Q}_{G}\left(w_\circ t_{-w_\circ\left(\alpha_i\right)}\right)}(-\varepsilon_{i+1})\right]\\ & +\sum_{\varnothing\neq\left\{i_1<\dotsm<i_a\right\}\subset\left\{1,\dotsc,i-1\right\}}\left(-1\right)^a \left[\mathcal{O}_{\mathbf{Q}_{G}\left(\left(i_1\dotsm i_ai\right)^{-1}w_\circ t_{-w_\circ\left(\alpha_{i_1,i-1}\right)}\right)}(-\varepsilon_{i})\right]\notag\\ & +\sum_{\varnothing\neq\left\{j_1<\dotsm<j_{b}\right\}\subset \left\{i+1,n+1\right\}} \left(-1\right)^{b-1}q\cdot\left[\mathcal{O}_{\mathbf{Q}_{G}\left(\left(i j_1\dotsm j_b\right)^{-1}w_\circ t_{-w_\circ\left(\alpha_{i,j_b-1}\right)}\right)}\left(-\varepsilon_{j_b}\right)\right],\notag \end{align} $$

where $1\le i\le n+1$ and

(A.8) $$ \begin{align} \mathbf{1}_{\{i<n+1\}}=\begin{cases} 1 &\text{if}\ i<n+1,\\ 0 &\text{otherwise.}\end{cases} \end{align} $$

Remark A.3. The leading term $\left [\mathcal {O}_{\mathbf {Q}_{G}\left (w_\circ \right )}(-\varepsilon _i)\right ]$ in equation (A.7) recovers the ‘classical’ analogue of this formula in $K_H(G/B)$ , via the map $\mathrm {trunc}$ of formula (1.3), while the remaining terms give the explicit corrections in $K_{H\times \mathbb {C}^*}(\mathbf {Q}_{G})$ .

Remark A.4. Applying the diagram automorphism ‘ $-w_\circ $ ,’ which sends

$$ \begin{align*}\left[\mathcal{O}_{\mathbf{Q}_{G}\left(wt_\beta\right)}(\mu)\right]\mapsto\left[\mathcal{O}_{\mathbf{Q}_{G}\left(w_\circ ww_\circ t_{-w_\circ\left(\beta\right)}\right)}(-w_\circ\mu)\right],\end{align*} $$

and an equivariant scalar $e^\lambda \mapsto e^{-w_\circ \lambda }$ , one obtains a similar formula for $e^{-\varepsilon _{n+2-i}}\cdot \left [\mathcal {O}_{\mathbf {Q}_{G}\left (w_{\circ }\right )}\right ]$ in $K_{H\times \mathbb {C}^*}(\mathbf {Q}_{G})$ . By the observations in Section 3.1.4, the action of $e^\lambda $ on $K_{H\times \mathbb {C}^*}(\mathbf {Q}_{G})$ for any $\lambda \in P$ is then completely determined by iteration of these two formulas together with the $D_i$ for $i\in I$ .

A.5. Symmetrisation

The usual q-Toda difference operators are realised geometrically by the ‘spherical part’ of $\mathbb {K}$ , which is the free $\mathfrak {H}$ -submodule generated by $\left [\mathcal {O}_{\mathbf {Q}_{G}}\right ]$ . By [Reference Orr25, Corollary 5.3], the action (of a spherical nil-affine Hecke subalgebra of $\mathbb {H}_0$ ) on the spherical part is obtained by taking the $(e,e)$ -entry of the corresponding matrix in $\mathrm {Mat}_W(\mathfrak {H})$ , namely

(A.9) $$ \begin{align} f(X)\cdot\left[\mathcal{O}_{\mathbf{Q}_{G}}\right]=\left[\mathcal{O}_{\mathbf{Q}_{G}}\right]\cdot\varrho_0(f)_{e,e}, \end{align} $$

for any symmetric Laurent polynomial $f=f(X)\in \mathbb {Z}\left [q^{\pm 1}\right ][X]^W\subset \mathbb {H}_0$ .

Applying equation (A.9) for $G=SL(n+1,\mathbb {C})$ to the symmetrisation of $e^{\varpi _1}$ , one obtains

(A.10) $$ \begin{align} \sum_{i=1}^{n+1}e^{\varepsilon_i}\cdot\left[\mathcal{O}_{\mathbf{Q}_{G}}\right]=\left[\mathcal{O}_{\mathbf{Q}_{G}}\right]\cdot\varrho_0\left(\sum_{i=1}^{n+1}X^{-\varepsilon_i}\right)_{e,e}, \end{align} $$

and we can use equation (3.62) to compute the right-hand side. One can easily show that just a single term contributes to the $(e,e)$ -entry for each $i=1,\dotsc ,n+1$ . The contributing terms are those given by the stationary walk $(e,\dotsc ,e)\in \mathbf {QW}^{(i-1)}_e$ . Thus equation (3.62) immediately results in the following:

(A.11) $$ \begin{align} \sum_{i=1}^{n+1}e^{\varepsilon_i}\cdot\left[\mathcal{O}_{\mathbf{Q}_{G}}\right]=\left[\mathcal{O}_{\mathbf{Q}_{G}}\right]\cdot\left(X^{-w_\circ\varepsilon_1}+\sum_{i=2}^{n+1}X^{-w_\circ\varepsilon_i}\left(1-q\cdot t_{-w_\circ\alpha_{i-1}} X^{-w_\circ\alpha_{i-1}}\right)\right). \end{align} $$

The element of $\mathfrak {H}$ acting on the right-hand side of equation (A.11) is an equivalent form of the usual first-order q-Toda difference operator (compare [Reference Givental and Lee5, equation (2)]).

Appendix B Proofs of Lemma 3.8 and Proposition 3.9

Recall the setting of Section 3.3.3. Equation (3.55) follows from the formula

(B.1) $$ \begin{align} \rho-w^{-1}\rho = \sum_{\alpha \in \mathrm{Inv} \left(w^{-1}\right)} \alpha \quad \text{for}\ w \in W, \end{align} $$

applied to the cases $w = x$ and $w=\left \lfloor w_{\circ } \right \rfloor $ , by using the fact that $(\alpha ,\varpi _{k})=1$ for all $\alpha \in \mathrm {Inv} \left (w^{-1}\right ) \subset \Delta ^{+} \setminus \Delta _{J}^{+}$ with $w \in W^{J}$ . Equation (3.56) follows from equation (B.1), using the fact that $(\lambda ,\gamma ) \in \{0,\pm 1\}$ for all $\gamma \in \Delta $ .

The following lemma is well known; indeed, it follows from the Cauchy – Schwarz inequality:

Lemma B.1. For $\alpha ,\beta \in \Delta $ , we have $(\alpha ,\beta ) \in \bigl \{ 0, \pm 1, \pm 2\bigr \}$ . In addition, $(\alpha ,\beta ) = \pm 2$ if and only if $\alpha =\pm \beta $ .

Lemma B.2.

  1. 1. $(\lambda ,\beta _{r}) = -1$ for all $1 \le r \le l$ .

  2. 2. $(\beta _{r},\beta _{t}) \in \bigl \{0,1\bigr \}$ for all $1 \le r$ , $t \le l$ with $r \ne t$ .

Proof. (1) We see that

$$ \begin{align*} (\lambda,\beta_{r}) & = \left(s_{j_{l}}s_{j_{l-1}} \cdots s_{j_{2}}s_{j_{1}}\varpi_{k}, s_{j_{l}}s_{j_{l-1}} \cdots s_{j_{r+1}}\left(\alpha_{j_{r}}\right) \right) \\ & = \Big(\underbrace{ s_{j_{r}}s_{j_{r-1}} \cdots s_{j_{2}}s_{j_{1}} }_{=:v} \varpi_{k},\,\alpha_{j_{r}}\Big) = \left(\varpi_{k},v^{-1}\alpha_{j_{r}}\right). \end{align*} $$

Since $v=s_{j_{r}}s_{j_{r-1}} \cdots s_{j_{2}}s_{j_{1}} \in W^{J}$ with $s_{j_{r}}v < v$ , it follows that $v^{-1}\alpha _{j_{r}} \in -\left (\Delta ^{+} \setminus \Delta _{J}^{+}\right )$ . Therefore, we have $\left (\varpi _{k},v^{-1}\alpha _{j_{r}}\right ) < 0$ ; since $\varpi _{k}$ is minuscule, we obtain $\left (\varpi _{k},v^{-1}\alpha _{j_{r}}\right ) = -1$ , as desired.

(2) By Lemma B.1, we have $(\beta _{r},\beta _{t}) \in \bigl \{0,\pm 1\bigr \}$ . Suppose, for a contradiction, that $(\beta _{r},\beta _{t}) = -1$ . Then $s_{\beta _{r}}(\beta _{t}) = \beta _{r}+\beta _{t} \in \Delta $ . By (1), we have $(\lambda ,\beta _{r}+\beta _{t}) = -2$ , which contradicts the assumption that $\varpi _{k}$ is minuscule. This proves the lemma.

Equation (3.57) follows from Lemma B.2(1). Equation (3.58) follows from equation (3.57) using the Dynkin diagram automorphism $\omega :I \rightarrow I$ induced by $w_{\circ }$ (see Appendix B.2).

In Appendices B.1 and B.2, we provide a complete proof of Proposition 3.9, since we do not know of a suitable reference for these assertions.

B.1. Proof of equation (3.59)

For $\alpha = \sum _{i \in I}c_{i}\alpha _{i} \in Q$ , we set $\mathrm {ht}(\alpha ):=\sum _{i \in I} c_{i} \in \mathbb {Z}$ .

Lemma B.3. Set $1 \le r < t \le l$ . If $(\beta _{r},\beta _{t})=1$ , then $\mathrm {ht}(\beta _{r})> \mathrm {ht}(\beta _{t})$ .

Proof. We prove the assertion by induction on $l=\ell (x)$ . If $l=0$ or $l=1$ , then the assertion is obvious. Assume that $l> 1$ . If $t = l$ , then $\beta _{t} = \beta _{l} = \alpha _{j_{l}}$ , and

$$ \begin{align*} s_{j_{l-1}} \cdots s_{j_{r+1}}\left(\alpha_{j_{r}}\right) = s_{j_{l}} \beta_{r} = s_{\beta_{t}} \beta_{r} = \beta_{r}-\beta_{t}. \end{align*} $$

Hence $\beta _{r} = \beta _{t} + s_{j_{l-1}} \cdots s_{j_{r+1}}\left (\alpha _{j_{r}}\right )$ ; note that $s_{j_{l-1}} \cdots s_{j_{r+1}}\left (\alpha _{j_{r}}\right ) \in \Delta ^{+}$ , since $s_{j_{l-1}} \cdots s_{j_{r+1}}s_{j_{r}}$ is reduced. Therefore, we obtain $\mathrm {ht}(\beta _{r})> \mathrm {ht}(\beta _{t})$ .

Assume that $t \le l-1$ . Note that $x':=s_{j_{l}}x \in W^{J}$ with $\ell (x') = \ell (x)-1$ , and that $x'=s_{j_{p-1}} \cdots s_{j_{2}}s_{j_{1}}$ is a reduced expression for $x'$ . We set

$$ \begin{align*} \beta_{u}':=s_{j_{l-1}} \cdots s_{j_{u+1}}\left(\alpha_{j_{u}}\right) = s_{j_{l}}\beta_{u} \quad \text{for}\ 1 \le u \le l-1. \end{align*} $$

Since $1 \le r < t \le l-1$ satisfy the condition that $\left (\beta _{r}',\beta _{t}'\right ) = (\beta _{r},\beta _{t})=1$ , it follows by our induction hypothesis that $\mathrm {ht}\left (\beta _{r}'\right )> \mathrm {ht}\left (\beta _{t}'\right )$ . Here we remark that

$$ \begin{align*} \beta_{r} = \beta_{r}' - \underbrace{\left(\beta_{r}',\alpha_{j_{l}}\right)}_{=:a} \alpha_{j_{l}}, \qquad \beta_{t} = \beta_{t}' - \underbrace{\left(\beta_{t}',\alpha_{j_{l}}\right)}_{=:b} \alpha_{j_{l}}, \end{align*} $$

where $a,b \in \{0,-1\}$ by Lemma B.2(2). If $a=-1$ or if $a=b=0$ , then it is obvious that $\mathrm {ht}(\beta _{r})> \mathrm {ht}(\beta _{t})$ , since $\mathrm {ht}\left (\beta ^{\prime }_{r}\right )> \mathrm {ht}\left (\beta ^{\prime }_{t}\right )$ by our induction hypothesis. Assume that $a=0$ and $b=-1$ . Suppose, for a contradiction, that $\mathrm {ht}(\beta _{r}) \le \mathrm {ht}(\beta _{t})$ . Since $\mathrm {ht}(\beta _{t}) = \mathrm {ht}\left (\beta ^{\prime }_{t}\right ) + 1$ and $\mathrm {ht}(\beta _{r}) = \mathrm {ht}\left (\beta ^{\prime }_{r}\right )$ , and since $\mathrm {ht}\left (\beta ^{\prime }_{r}\right )> \mathrm {ht}\left (\beta ^{\prime }_{t}\right )$ by our induction hypothesis, we have $\mathrm {ht}(\beta _{r}) = \mathrm {ht}(\beta _{t})$ . Since $(\beta _{r},\beta _{t})=1$ by the assumption, we see that $\beta _{r}-\beta _{t} \in \Delta $ . However, $\mathrm {ht}(\beta _{r}-\beta _{t}) =\mathrm {ht}(\beta _{r})-\mathrm {ht}(\beta _{t})=0$ , which is a contradiction. Hence we conclude that $\mathrm {ht}(\beta _{r})> \mathrm {ht}(\beta _{t})$ , as desired. This proves the lemma.□

Lemma B.4. Set $1 \le r \le l$ , and set $h:=\mathrm {ht}(\beta _{r})$ . Then

(B.2) $$ \begin{align} & \# \{ r < t \le l \mid (\beta_{t},\beta_{r})=1\}=h-1, \end{align} $$

(B.3) $$ \begin{align} & \# \left\{ \alpha \in \Delta^{+} \mid \mathrm{ht}(\alpha) < \mathrm{ht}(\beta_{r}), (\alpha,\beta_{r})=1 \right\} = 2(h-1). \end{align} $$

Proof. We prove the assertion by induction on $l=\ell (x)$ . If $l=0$ or $l=1$ , then the assertion is obvious. Assume that $l> 1$ . If $r = l$ , then the assertion is obvious. Assume now that $1 \le r \le l-1$ . Note that $x':=s_{j_{l}}x \in W^{J}$ with $\ell (x') = \ell (x)-1$ , and that $x'=s_{j_{l-1}} \cdots s_{j_{2}}s_{j_{1}}$ is a reduced expression for $x'$ . We set

$$ \begin{align*} \beta_{u}':=s_{j_{l-1}} \cdots s_{j_{u+1}}\left(\alpha_{j_{u}}\right) = s_{j_{l}}\beta_{u} \quad \text{for}\ 1 \le u \le l-1. \end{align*} $$

Since $\beta _{l}=\alpha _{j_{l}}$ , we see by Lemma B.2(2) that $\left (\beta _{r}', \alpha _{j_{l}}\right ) = - \left (\beta _{r},\alpha _{j_{l}}\right ) = - (\beta _{r},\beta _{l}) \in \{0,-1\}$ .

Case 1. Assume first that $\left (\beta _{r}', \alpha _{j_{l}}\right ) = 0$ , or equivalently, $\left (\beta _{r}, \alpha _{j_{l}}\right ) = 0$ ; in this case, $\beta _{r}=\beta _{r}'$ , and hence $\mathrm {ht}\left (\beta _{r}'\right )=\mathrm {ht}(\beta _{r})=h$ . In addition, since $\beta _{l}=\alpha _{j_{l}}$ , it follows that

$$ \begin{align*} \{ r < t \le l \mid (\beta_{t},\beta_{r})=1\} & = \{ r < t \le l-1 \mid (\beta_{t},\beta_{r})=1\} \\ & = \left\{ r < t \le l-1 \mid \left(s_{j_{l}}\beta_{t},s_{j_{l}}\beta_{r}\right)=1\right\} \\ & = \left\{ r < t \le l-1 \mid \left(\beta_{t}',\beta_{r}'\right)=1\right\}\!. \end{align*} $$

By our induction hypothesis, we have $\# \left \{ r < t \le l-1 \mid \left (\beta _{t}',\beta _{r}'\right )=1\right \} = h-1$ , and hence $\# \{ r < t \le l \mid (\beta _{t},\beta _{r})=1\} =h-1$ , as desired. Also, since $\beta _{r}=\beta _{r}'$ in this case, it is obvious that

$$ \begin{align*} & \left\{ \alpha \in \Delta^{+} \mid \mathrm{ht}(\alpha) < \mathrm{ht}(\beta_{r}), (\alpha,\beta_{r})=1\right\} = \left\{ \alpha \in \Delta^{+} \mid \mathrm{ht}(\alpha) < \mathrm{ht}\left(\beta^{\prime}_{r}\right), \left(\alpha,\beta^{\prime}_{r}\right)=1\right\}\!. \end{align*} $$

Since $\# \left \{ \alpha \in \Delta ^{+} \mid \mathrm {ht}(\alpha ) < \mathrm {ht}\left (\beta ^{\prime }_{r}\right ), \left (\alpha ,\beta ^{\prime }_{r}\right )=1\right \}= 2(h-1)$ by our induction hypothesis, we obtain equation (B.3), as desired.

Case 2. Assume next that $\left (\beta ^{\prime }_{r}, \alpha _{j_{l}}\right ) = -1$ , or equivalently, $\left (\beta _{r}, \alpha _{j_{l}}\right ) = 1$ ; in this case, $\beta _{r}=\beta ^{\prime }_{r}+\alpha _{j_{l}}$ , and hence $\mathrm {ht}\left (\beta ^{\prime }_{r}\right )=\mathrm {ht}(\beta _{r})-1=h-1$ . In addition, since $\beta _{l}=\alpha _{j_{l}}$ , it follows that

$$ \begin{align*} \{ r < t \le l \mid (\beta_{t},\beta_{r})=1\} & = \{ r < t \le l-1 \mid (\beta_{t},\beta_{r})=1\} \cup \{l\} \\ & = \left\{ r < t \le l-1 \mid \left(s_{j_{l}}\beta_{t},s_{j_{l}}\beta_{r}\right)=1\right\} \cup \{l\} \\ & = \left\{ r < t \le l-1 \mid \left(\beta_{t}',\beta_{r}'\right)=1\right\} \cup \{l\}. \end{align*} $$

By our induction hypothesis, we have $\# \left \{ r < t \le l-1 \mid \left (\beta _{t}',\beta _{r}'\right )=1\right \} = h-2$ , and hence $\# \{ r < t \le l \mid (\beta _{t},\beta _{r})=1\} = h-1$ , as desired.

Let us prove equation (B.3) in this case. For simplicity of notation, we set

(B.4) $$ \begin{align} & R=R_{r}:= \left\{ \alpha \in \Delta^{+} \mid \mathrm{ht}(\alpha) < \mathrm{ht}(\beta_{r}), (\alpha,\beta_{r})=1\right\}\kern-2pt, \nonumber\\ & S=S_{r}:= \left\{ \alpha \in \Delta^{+} \mid \mathrm{ht}(\alpha) < \mathrm{ht}\left(\beta^{\prime}_{r}\right), \left(\alpha,\beta^{\prime}_{r}\right)=1\right\}\kern-2pt. \end{align} $$

Since $\left (\beta _{r}, \alpha _{j_{l}}\right ) = 1$ and $\beta _{r}-\alpha _{j_{l}} = s_{j_{l}}\beta _{r} = \beta ^{\prime }_{r} \in \Delta ^{+}$ , we deduce that $\alpha _{j_{l}},\,\beta _{r}-\alpha _{j_{l}} \in R$ . We claim that

(B.5) $$ \begin{align} s_{j_{l}}\alpha \in S \quad \text{for all}\ \alpha \in R \setminus \left\{ \alpha_{j_{l}}, \beta_{r}-\alpha_{j_{l}} \right\}\kern-2pt. \end{align} $$

It is easily verified that $s_{j_{l}}\alpha \in \Delta ^{+}$ and $\left (s_{j_{l}}\alpha ,\beta ^{\prime }_{r}\right ) = 1$ . We show that $\mathrm {ht}\left (s_{j_{l}} \alpha \right ) < \mathrm {ht}\left (\beta ^{\prime }_{r}\right )$ . By Lemma B.1, $\left (\alpha ,\alpha _{j_{l}}\right ) \in \{0,\pm 1\}$ . If $\left (\alpha ,\alpha _{j_{l}}\right ) = 1$ , then it is obvious that $\mathrm {ht}\left (s_{j_{l}} \alpha \right ) < \mathrm {ht}\left (\beta ^{\prime }_{r}\right )$ . Assume that $\left (\alpha ,\alpha _{j_{l}}\right ) = 0$ . Suppose, for a contradiction, that $\mathrm {ht}\left (s_{j_{l}} \alpha \right ) \ge \mathrm {ht}\left (\beta ^{\prime }_{r}\right )$ . Since $\mathrm {ht}(\alpha ) < \mathrm {ht}(\beta _{r})$ and $\mathrm {ht}\left (\beta ^{\prime }_{r}\right ) = \mathrm {ht}(\beta _{r}) -1$ , we have $\mathrm {ht}\left (s_{j_{l}} \alpha \right ) = \mathrm {ht}\left (\beta ^{\prime }_{r}\right )$ . Since $\left (s_{j_{l}} \alpha ,\beta ^{\prime }_{r}\right )=1$ , we see that $\beta ^{\prime }_{r} - s_{j_{l}} \alpha \in \Delta $ . However, $\mathrm {ht}\left (\beta ^{\prime }_{r} - s_{j_{l}} \alpha \right ) =0$ , which is a contradiction. Assume that $\left (\alpha ,\alpha _{j_{l}}\right ) = -1$ ; in this case, $s_{j_{l}}\alpha = \alpha +\alpha _{j_{l}}$ . Suppose, for a contradiction, that $\mathrm {ht}\left (s_{j_{l}}\alpha \right ) \ge \mathrm {ht}\left (\beta ^{\prime }_{r}\right )$ ; since $\mathrm {ht}\left (s_{j_{l}}\alpha \right ) = \mathrm {ht}(\alpha )+1$ and $\mathrm {ht}\left (\beta ^{\prime }_{r}\right ) = \mathrm {ht}(\beta _{r})-1$ , and since $\mathrm {ht}(\alpha ) < \mathrm {ht}(\beta _{r})$ , $\mathrm {ht}\left (s_{j_{l}}\alpha \right )$ is equal to $\mathrm {ht}\left (\beta ^{\prime }_{r}\right )$ or $\mathrm {ht}\left (\beta ^{\prime }_{r}\right )+1$ . By the same reasoning as before, we see that $\mathrm {ht}\left (s_{j_{l}}\alpha \right ) \ne \mathrm {ht}\left (\beta ^{\prime }_{r}\right )$ . Therefore, $\mathrm {ht}\left (s_{j_{l}}\alpha \right ) = \mathrm {ht}\left (\beta ^{\prime }_{r}\right )+1$ , and hence $\mathrm {ht}(\alpha )=\mathrm {ht}(\beta _{r})-1$ . Also, we see that $s_{\alpha }\beta _{r} = \beta _{r}-\alpha \in \Delta ^{+}$ . From these, we deduce that $\alpha = \beta _{r}-\alpha _{j}$ for some $j \in I$ . It follows that

$$ \begin{align*} -1 = \left(\alpha,\alpha_{j_{l}}\right) = \left(\beta_{r},\alpha_{j_{l}}\right)- \left(\alpha_{j},\alpha_{j_{l}}\right) = 1 - \left(\alpha_{j},\alpha_{j_{l}}\right)\kern-2pt, \end{align*} $$

and hence $\left (\alpha _{j},\alpha _{j_{l}}\right ) = 2$ . This implies that $j = j_{l}$ , and hence $\alpha = \beta _{r}-\alpha _{j_{l}}$ , which is a contradiction. Thus, we have shown that $\mathrm {ht}\left (s_{j_{l}} \alpha \right ) < \mathrm {ht}\left (\beta ^{\prime }_{r}\right )$ in all cases. Therefore, we obtain a map

$$ \begin{align*} s_{j_{l}} : R \setminus \left\{ \alpha_{j_{l}}, \beta_{r}-\alpha_{j_{l}} \right\} \rightarrow S, \qquad \alpha \mapsto s_{j_{l}}\alpha. \end{align*} $$

Similarly, we obtain its inverse

$$ \begin{align*} s_{j_{l}} : S \rightarrow R \setminus \left\{ \alpha_{j_{l}}, \beta_{r}-\alpha_{j_{l}} \right\}, \qquad \alpha \mapsto s_{j_{l}}\alpha. \end{align*} $$

Recall that $\mathrm {ht}\left (\beta ^{\prime }_{r}\right )=\mathrm {ht}(\beta _{r})-1=h-1$ . We conclude that

$$ \begin{align*} \# R & = \# \left(R \setminus \left\{ \alpha_{j_{l}}, \beta_{r}-\alpha_{j_{l}} \right\}\right) + 2 \\ & = \# S + 2 = 2(h-2) + 2 = 2(h-1), \end{align*} $$

as desired. This proves the lemma.□

Remark B.5. Keep the notation and setting as before. Set $1 \le r \le l$ . We set

(B.6) $$ \begin{align} R_{r}:= \left\{ \alpha \in \Delta^{+} \mid \mathrm{ht}(\alpha) < \mathrm{ht}(\beta_{r}), (\alpha,\beta_{r})=1\right\}\kern-2pt. \end{align} $$

If $\alpha \in R_{r}$ , then $s_{\alpha }\beta _{r} = \beta _{r}-\alpha \in \Delta ^{+}$ ; it is easily verified that $\beta _{r}-\alpha \in R_{r}$ . Hence $\sigma :\alpha \mapsto \beta _{r}-\alpha $ is an involution on $R_{r}$ ; notice that $\sigma $ is fixed-point-free (otherwise, $\beta _{r}=2\alpha $ for some $\alpha \in R_{r}$ ). By Lemma B.3,

$$ \begin{align*} B_{r}:= \{ \beta_{t} \mid r < t \le l,\,(\beta_{t},\beta_{r})=1\} \end{align*} $$

is a subset of $R_{r}$ ; recall from Lemma B.4 that $\#B_{r}=h-1$ and $\#R_{r}=2(h-1)$ , where $h:=\mathrm {ht}(\beta _{r})$ . We claim that $\sigma (B_{r}) \sqcup B_{r} = R_{r}$ . Suppose, for a contradiction, that $\sigma (\beta _{t}) = \beta _{r}-\beta _{t} \in B_{r}$ for some $\beta _{t} \in B_{r}$ . Let $r < s \le l$ be such that $\sigma (\beta _{t}) = \beta _{s}$ . Then $\beta _{r} = \beta _{s}+\beta _{t}$ , and hence $(\lambda ,\beta _{r}) = (\lambda , \beta _{s})+(\lambda ,\beta _{t}) = -2$ by Lemma B.2(1). However, this contradicts the assumption that $\lambda $ is minuscule. It follows that $\sigma (B_{r}) \cap B_{r} = \emptyset $ , and hence $\# ( \sigma (B_{r}) \cup B_{r} ) = 2(h-1) = \# R_{r}$ . Thus, we conclude that $\sigma (B_{r}) \sqcup B_{r} = R_{r}$ .

We will prove equation (3.59). For $v \in \left \{w,\,s_{\beta _{r}}w\right \}$ , we set

$$ \begin{align*} X(v) & := \left\{ \gamma \in \mathrm{Inv}(v) \mid s_{\beta_{r}}\gamma \in \Delta^{+} \right\}\kern-2pt, \\ Y(v) & := \left\{ \gamma \in \mathrm{Inv}(v) \mid s_{\beta_{r}}\gamma \in \Delta^{-} \right\}\kern-2pt. \end{align*} $$

Note that $\mathrm {Inv} (v) = X(v) \sqcup Y(v)$ for $v \in \left \{w,s_{\beta _{r}}w\right \}$ . We see that the map $X(w) \rightarrow X\left (s_{\beta _{r}}w\right )$ , $\gamma \mapsto s_{\beta _{r}}\gamma $ , is bijective. Therefore,

$$ \begin{align*} \ell(w)-\ell\left(s_{\beta_{r}}w\right) = \# Y(w) - \# Y\left(s_{\beta_{r}}w\right). \end{align*} $$

Also, by Lemma B.1 and the assumption that $s_{\beta _{r}}w < w$ , it follows that

$$ \begin{align*} Y(w) = \{ \beta_{r} \} \sqcup (\mathrm{Inv} (w) \cap R_{r}), \qquad Y\left(s_{\beta_{r}} w\right) = \mathrm{Inv}\left(s_{\beta_{r}}w\right) \cap R_{r} \end{align*} $$

(for the definition of $R_{r}$ , see Remark B.5). Recall from Remark B.5 that for $\beta _{t} \in B_{r}$ , $\sigma (\beta _{t}) = \beta _{r}-\beta _{t} \in R_{r} \setminus B_{r}$ , and that $R_{r}=B_{r} \sqcup \sigma (B_{r})$ . Since $\beta _{r} \in \mathrm {Inv}(w)$ , for each $\beta _{t} \in B_{r}$ , one of the following holds:

  1. (i) $\beta _{t},\sigma (\beta _{t}) \in \mathrm {Inv}(w)$ .

  2. (ii) $\beta _{t} \in \mathrm {Inv}(w)$ and $\sigma (\beta _{t}) \not \in \mathrm {Inv}(w)$ .

  3. (iii) $\beta _{t} \not \in \mathrm {Inv}(w)$ and $\sigma (\beta _{t}) \in \mathrm {Inv}(w)$ .

We see that these are equivalent, respectively, to the following:

  1. (i)’ $\beta _{t},\sigma (\beta _{t}) \not \in \mathrm {Inv}\left (s_{\beta _{r}}w\right )$ .

  2. (ii)’ $\beta _{t} \in \mathrm {Inv}\left (s_{\beta _{r}}w\right )$ and $\sigma (\beta _{t}) \not \in \mathrm {Inv}\left (s_{\beta _{r}}w\right )$ .

  3. (iii)’ $\beta _{t} \not \in \mathrm {Inv}\left (s_{\beta _{r}}w\right )$ and $\sigma (\beta _{t}) \in \mathrm {Inv}\left (s_{\beta _{r}}w\right )$ .

Therefore, if we set

$$ \begin{align*} & a:=\# \{ \beta_{t} \in B_{r} \mid \beta_{t},\sigma(\beta_{t}) \in \mathrm{Inv}(w) \}, \\ & b:=\# \{ \beta_{t} \in B_{r} \mid \beta_{t} \in \mathrm{Inv}(w), \sigma(\beta_{t}) \not\in \mathrm{Inv}(w) \}, \\ & c:=\# \{ \beta_{t} \in B_{r} \mid \beta_{t} \notin \mathrm{Inv}(w), \sigma(\beta_{t}) \in \mathrm{Inv}(w) \}, \end{align*} $$

we have

$$ \begin{align*} \# Y(w) & = 1 + \#(\mathrm{Inv} (w) \cap R_{r}) = 1 + 2a + b + c, \\ \# Y\left(s_{\beta_{r}}w\right) & = \#\left(\mathrm{Inv} \left(s_{\beta_{r}}w\right) \cap R_{r}\right) = b + c, \end{align*} $$

and hence

(B.7) $$ \begin{align} \ell(w)-\ell\left(s_{\beta_{r}}w\right) = \# Y(w) - \# Y\left(s_{\beta_{r}}w\right) = 1+2a. \end{align} $$

Now we see that

$$ \begin{align*} & \ell_{\lambda,r}^{-}(w)-\ell_{\lambda,r}^{-}\left(s_{\beta_{r}}w\right) \\ & \quad = \# ( \mathrm{Inv}(w) \cap \{\beta_{r},\beta_{r+1},\dots,\beta_{l}\} ) - \# \left( \mathrm{Inv}\left(s_{\beta_{r}}w\right) \cap \{\beta_{r},\beta_{r+1},\dots,\beta_{l}\} \right) \\ & \quad = \# (\{ \beta_{r} \} \sqcup (\mathrm{Inv}(w) \cap \{\beta_{r+1},\dots,\beta_{l}\} )) - \# \left( \mathrm{Inv}\left(s_{\beta_{r}}w\right) \cap \{\beta_{r+1},\dots,\beta_{l}\} \right)=:(\ast), \end{align*} $$

where the last equality follows from the assumption that $s_{\beta _{r}}w < w$ . Also, we deduce that for $r < s \le l$ with $(\beta _{s}, \beta _{r})=0$ , $\beta _{s} \in \mathrm {Inv}(w)$ if and only if $\beta _{s} \in \mathrm {Inv}\left (s_{\beta _{r}}w\right )$ . Therefore,

$$ \begin{align*} (\ast) & = 1 + \# (\mathrm{Inv}(w) \cap B_{r}) - \# \left(\mathrm{Inv}\left(s_{\beta_{r}}w\right) \cap B_{r}\right) \\ & = 1 + (a+b) - b = 1+a. \end{align*} $$

Thus we conclude that

$$ \begin{align*} 2 \left( \ell_{\lambda,r}^{-}(w)-\ell_{\lambda,r}^{-}\left(s_{\beta_{r}}w\right) \right) - 1 =2 ( 1+a ) - 1 = 2a+1 \stackrel{(B.7)}{=} \ell(w)-\ell\left(s_{\beta_{r}}w\right), \end{align*} $$

as desired. This proves equation (3.59).

B.2. Proofs of equations (3.58) and (3.60)

Let $\omega :I \rightarrow I$ be the Dynkin diagram automorphism induced by the longest element $w_{\circ }$ – that is, $w_{\circ } \alpha _{i}= -\alpha _{\omega (i)}$ for $i \in I$ ; note that $\left \lfloor w_{\circ } \right \rfloor \varpi _{k} = w_{\circ } \varpi _{k} = - \varpi _{\omega (k)}$ . We see that $\varpi _{\omega (k)}$ is also minuscule. In addition, $y^{-1}=s_{i_{m}} \cdots s_{i_{1}} \in W^{\omega (J)} = W^{I \setminus \{\omega (k)\}}$ . We set $\nu :=y^{-1}\varpi _{\omega (k)}$ ; for $\gamma \in \Delta ^{+}$ ,

$$ \begin{align*} (\gamma,\lambda)=1 \iff \left(\gamma,y^{-1}\left\lfloor w_{\circ} \right\rfloor\varpi_{k}\right)=1 \iff \left(\gamma,y^{-1}\left(-\varpi_{\omega(k)}\right)\right)=1 \iff (\gamma,\nu) = -1. \end{align*} $$

Hence it follows that $\mathrm {Inv}(w)_{\lambda }^{+} = \mathrm {Inv}(w)_{\nu }^{-}$ for $w \in W$ . Also, it is easily verified that $\ell _{\lambda ,s}^{+}(w) = \ell _{\nu ,s}^{-}(w)$ for all $w \in W$ . Thus equations (3.58) and (3.60) follow from equations (3.57) and (3.59) (applied to $\nu $ ), respectively.

This completes the proof of Proposition 3.9.

Acknowledgements

The authors would like to thank Cristian Lenart, Leonardo Mihalcea and Mark Shimozono for helpful discussions. We also thank an anonymous referee for useful suggestions which improved our exposition. The first author was supported in part by Grant-in-Aid for JSPS Fellows 20J12058. The second author was supported in part by JSPS Grant-in-Aid for Scientific Research (B) 16H03920. The third author was supported in part by a Collaboration Grant for Mathematicians from the Simons Foundation (Award ID: 638577). The fourth author was supported in part by JSPS Grant-in-Aid for Scientific Research (C) 19K03415.

Conflict of Interest

None.

Footnotes

1 Provided their existence, the coefficients $c_{x,y}^{\lambda ,\mu }, d_{x,y}^{\lambda ,\mu }\in \mathbb {Z}\left [q^{\pm 1}\right ]$ are unique, since both $\left \{e^\lambda \cdot \left [\mathcal {O}_{\mathbf {Q}_{G}(x)}\right ]\right \}_{x\in W_{\mathrm {af}}^{\ge 0},\lambda \in P}$ and $\left \{\left [\mathcal {O}_{\mathbf {Q}_{G}(x)}(\lambda )\right ]\right \}_{x\in W_{\mathrm {af}}^{\ge 0},\lambda \in P}$ are (topologically) $\mathbb {Z}\left [q^{\pm 1}\right ]$ -linearly independent sets in $K_{H\times \mathbb {C}^*}(\mathbf {Q}_{G})$ . Existence follows from [Reference Kato, Naito and Sagaki13Reference Naito, Orr and Sagaki24] for equation (1.1) and [Reference Orr25] for equation (1.2).

References

Braverman, A. and Finkelberg, M., ‘Semi-infinite Schubert varieties and quantum $K$ -theory of flag manifolds’, J. Amer. Math. Soc. 27(4) (2014), 11471168.CrossRefGoogle Scholar
Braverman, A. and Finkelberg, M., ‘Weyl modules and $q$ -Whittaker functions’, Math. Ann. 359(1-2) (2014), 4559.CrossRefGoogle Scholar
Brenti, F., Fomin, S. and Postnikov, A., ‘Mixed Bruhat operators and Yang-Baxter equations for Weyl groups’, Internat. Math. Res. Notices 1999 (8), 419441.CrossRefGoogle Scholar
Cherednik, I. and Orr, D., ‘Nonsymmetric difference Whittaker functions’, Math. Z. 279(3-4) (2015), 879938.CrossRefGoogle Scholar
Givental, A. and Lee, Y.-P., ‘Quantum $K$ -theory on flag manifolds, finite-difference Toda lattices and quantum groups’, Invent. Math. 151(1) (2003), 193219.CrossRefGoogle Scholar
Green, R. M., Combinatorics of Minuscule Representations, Cambridge Tracts in Mathematics vol. 199 (Cambridge University Press, Cambridge, 2013).Google Scholar
Griffeth, S. and Ram, A., ‘Affine Hecke algebras and the Schubert calculus’, European J. Combin. 25 (2004), 12631283.CrossRefGoogle Scholar
Kostant, B. and Kumar, S., ‘ $T$ -equivariant $K$ -theory of generalized flag varieties’, J. Differential Geom. 32(2) (1990), 549603.CrossRefGoogle Scholar
Kac, V. G., Infinite Dimensional Lie Algebras, 3rd edn (Cambridge University Press, Cambridge, 1990).CrossRefGoogle Scholar
Kato, S., ‘Loop structure on equivariant $K$ -theory of semi-infinite flag manifolds’, Preprint, 2020, arXiv:1805.01718.CrossRefGoogle Scholar
Kato, S., ‘Frobenius splitting of Schubert varieties of semi-infinite flag manifolds’, Preprint, 2021, arXiv:1810.07106.Google Scholar
Kato, S., ‘On quantum $K$ -groups of partial flag manifolds’, Preprint, 2020, arXiv:1906.09343.Google Scholar
Kato, S., Naito, S. and Sagaki, D., ‘Equivariant $K$ -theory of semi-infinite flag manifolds and the Pieri-Chevalley formula’, Duke Math. J. 169 (2020), 24212500.CrossRefGoogle Scholar
Koroteev, P., ‘A-type quiver varieties and ADHM moduli spaces, Comm. Math. Phys. 381(1) (2021), 175207.CrossRefGoogle Scholar
Koroteev, P. and Zeitlin, A. M., ‘qKZ/tRS duality via quantum K-theoretic counts’, Preprint, 2019, arXiv:1802.04463.Google Scholar
Kouno, T., Naito, S., Orr, D. and Sagaki, D., ‘Inverse $K$ -Chevalley formulas for semi-infinite flag manifolds, II’, In preparation.Google Scholar
Lenart, C., ‘A $K$ -theory version of Monk’s formula and some related multiplication formulas’, J. Pure Appl. Algebra 179(1-2) (2003), 137158.CrossRefGoogle Scholar
Lenart, C., ‘On the combinatorics of crystal graphs. II. The crystal commutor’, Proc. Amer. Math. Soc. 136(3) (2008), 825837.CrossRefGoogle Scholar
Lenart, C., Naito, S. and Sagaki, D., ‘A general Chevalley formula for semi-infinite flag manifolds and quantum $K$ -theory’, Preprint, 2021, arXiv:2010.06143.Google Scholar
Lenart, C. and Postnikov, A., ‘Affine Weyl groups in $K$ -theory and representation theory’, Int. Math. Res. Not. IMRN 2007(9) (2007), rnm038.Google Scholar
Lenart, C. and Shimozono, M., ‘Equivariant $K$ -Chevalley rules for Kac-Moody flag manifolds’, Amer. J. Math. 136 (2014), 11751213.CrossRefGoogle Scholar
Littelmann, P. and Seshadri, A., ‘Pieri-Chevalley type formula for $K\left(G/ B\right)$ and standard monomial theory’, in Studies in Memory of Issai Schur, Progress in Mathematics vol. 210 (Birkhäuser, Boston, MA, 2003), 155176.CrossRefGoogle Scholar
Mathieu, O., ‘Positivity of some intersections in ${K}_0\left(G/ B\right)$ ’, Commutative algebra, homological algebra and representation theory (Catania/Genoa/Rome, 1998), J. Pure Appl. Algebra 152(1-3) (2000), 231243.CrossRefGoogle Scholar
Naito, S., Orr, D. and Sagaki, D., ‘Chevalley formula for anti-dominant weights in the equivariant $K$ -theory of semi-infinite flag manifolds’, Adv. Math. 387 (2021), 107828.CrossRefGoogle Scholar
Orr, D., ‘Equivariant $K$ -theory of the semi-infinite flag manifold as a nil-DAHA module’, Preprint, 2020, arXiv:2001.03490.Google Scholar
Pittie, H. and Ram, A., ‘A Pieri-Chevalley formula in the $K$ -theory of a $G/ B$ -bundle’, Electron. Res. Announc. Amer. Math. Soc. 5 (1999), 102107.CrossRefGoogle Scholar
Stembridge, J. R., ‘Combinatorial models for Weyl characters’, Adv. Math. 168(1) (2002), 96131.CrossRefGoogle Scholar